You are on page 1of 112

31.

After having a mastectomy for the removal of an extensive breast cancer, the 44-year-old patient tells her physician that she has great difficulty abducting her arm above the level of her shoulder. Physical examination reveals that she has a "winged" scapula when she pushes against the wall. What structure has been injured during her breast surgery to result in this problem? A. Axillary nerve B. Pectoralis major muscle C. Deltoid muscle D. Long thoracic nerve E. Lateral pectoral nerve A 47-year-old male patient is scheduled to have a coronary arterial bypass because of chronic anginal pains. Coronary arteriography reveals nearly total blockage of the LAD artery. In exposing this artery to perform the bypass procedure, what accompanying vessel must be protected from injury? A. B. C. D. E. Middle cardiac vein Great cardiac vein Small cardiac vein Anterior cardiac vein Coronary venous sinus

32.

33.

As you examine the 14-year-old female patient from Portsmouth, you realize that the evidence on physical examination indicates clearly that she has a right-sided Bell's palsy. Which of the following signs would lead you to this conclusion? A. B. C. D. E. The patient's jaw muscles are so weak that she has difficulty chewing. She has no pain nor temperature sense in her right cheek. She is unable to close her right eye tightly. Her right pupil is constricted more than her left pupil. Her tongue deviates to the left when it is protruded.

34.

In preparation for delivery of the baby, the anesthesiologist elects to inject a suitable quantity of anesthetic into the CSF and then so position the patient that the anesthetic will have its principal effect upon the lower portion of the body including the pelvic region. Where should the anesthetic be injected to achieve the desired result? A. B. C. D. E. Into the epidural space at the sacral hiatus Into the extradural space at L3 Into the subarachnoid space at T12/L1 Into the subarachnoid space at L3/L4 Into the subdural space at Ll/L2

35.

During a fight in a tavern in Roseau, the 45-year-old construction worker received a shallow stab wound at a point near the middle of the left posterior triangle of the neck from a broken Kubuli bottle. On physical examination, it is noted that the left shoulder is drooping lower than the right shoulder, and the

superior angle of the left scapula juts out slightly. Strength in turning the head to the right or left appears symmetric. From these facts, what has been injured? A. B. C. D. E. 36. The origin of the left vagus nerve The site of termination of the left dorsal scapular nerve The anterior part of the left ansa cervicalis The proximal segment of the left phrenic nerve The left spinal accessory nerve, inferior to the sternocleidomastoid muscle

The 60-year-old woman had been experiencing chronic lancinating pain (very sharp, acute pain) between her left lower eyelid and her left upper lip. A diagnosis of tic douloureux is made and the decision is made to inject the ganglion wherein the highly irritable cell bodies are located which are responsible for the pain. Into which foramen should the needle be directed to reach the ganglion? A. B. C. D. E. The left infraorbital foramen The left stylomastoid foramen The left mental foramen The left jugular foramen The left supraorbital foramen

37.

The 47-year-old female patient's right breast exhibited peau d orange characteristics. This condition is primarily a result of A. B. C. D. E. blockage of cutaneous lymphatic vessels. shortening of the suspensory ligaments by cancer in the axillary tail of the breast. contraction of the retinacula cutis of the areola and nipple. invasion of the pectoralis major by metastatic cancer. ipsilateral (same side as the disease) inversion of the nipple from cancer of the duct system of the breast.

38.

The patient's chest had obviously suffered severe blunt trauma from impact with the steering wheel of the automobile when the auto hit the tree. Electrocardiographic data indicated that the heart had been injured by contact with the sternum. The MOST LIKELY part of the heart to be bruised by the force of impact with the sternum would be the ... A. B. C. D. E. conus region apex of left ventricle dextral margin of the right ventricle obtuse margin of the left ventricle anterior margin of the left atrium

39.

The 27-year-old male had received a small caliber bullet wound to the chest in the region of the third intercostal space, several centimeters to the left of the sternum. After reception in the Emergency Department in the large Miami hospital, a preliminary notation of "Beck's triad" was entered on the patient's chart. This comment referred to the fact that, among other possibilities of the injury that ... A. B. C. D. E. there is injury to the left pulmonary artery, left primary bronchus and esophagus. the patient has bleeding into the pleural cavity, a collapsed lung and mediastinal shift to the right. the patient has a small, quiet heart, decreased pulse pressure and increased central venous pressure. the young man is suffering from marked Diastolic emptying, Dyspnea, and Dilation of the aortic arch. the left lung has collapsed, there is paradoxical respiration, and there is a mediastinal shift of the heart and trachea to the left.

40.

The condition of aortic coarctation distal to the ductus arteriosus ... A. B. C. D. E. results in diversion of aortic blood such that arterial pressure in the lower limbs is higher than that in the upper limbs. is usually accompanied by a reduction of flow into the subclavian arteries. can result in "notching" of the ribs as seen on X-ray. is one of the features of the "Tetralogy of Fallot leads to death at birth if the ductus arteriosus fails to close.

41.

In a ventricular septal defect of the heart ... A. B. C. D. E. there is failure of closure of the septum primum. the endocardial cushions fuse with the aorticopulmonary septum. there is a shunting of blood from the right ventricle to the left ventricle. there is always a resulting stenosis of the pulmonary trunk. hypertrophy of the right ventricle occurs.

42.

The 29-year-old female patient exhibits gustatory sweating, several months after having her left parotid gland removed because of a benign tumor; that is, she sweats profusely from the ipsilateral (same side) of the face when she smells food that she enjoys, or even thinks of it. This problem... A. B. C. results from growth of fibers from the auriculotemporal nerve into the subcutaneous tissue in the area of the bed of the parotid gland. occurs because sweat glands are activated by adrenergic (epinephrine or norepinephrine) chemical mediators. results from regeneration of sympathetic fibers accompanying the arterial supply of the parotid gland.

D. E. 43.

could be corrected simply by cutting the vagus nerve on the affected side of the face. is one of the common side effects of Bell's palsy.

The orthopedic surgeon wanted to operate on the patient's hand without using general anesthesia. She recognized that she could inject anesthetic into the axillary connective tissue sheath containing the nerves of the upper limb and achieve a nice anesthetic condition of the hand, suitable for operation without causing the patient unnecessary pain or loss of consciousness. She knows she must exercise care, however, because paralysis of half of the diaphragm could occur if the anesthetic entered the space deep to the... A. B. C. D. E. prevertebral fascia anterior to the anterior scalene muscle. retropharyngeal fascia. superficial fascia enclosing the platysma muscle. alar fascia investing fascia enclosing the sternocleidomastoid and trapezius muscles.

44.

The 3-year-old child's eyelids had become swollen shut within a day after the dog had bitten her in the posterior part of her scalp. You, examining this suffering baby in the Emergency Department of the Staten Island Hospital in New York on your first evening of night call, know that ... A. B. C. D. E. the pain fibers from this injury are being carried by the dorsal primary ramus of the first cervical nerve. bleeding, and possibly infection, have spread through the areolar layer of the scalp. the "danger zone" of the scalp is localized to the space between the connective tissue of the scalp and the aponeurotic layer of the scalp. bleeding has spread through the space between the periosteum and the skull from the back of the head to the region of the eyes. the supratrochlear and supraorbital nerves must be anesthetized before anything else is done for the patient.

45.

The 35-year-old patient has a hard nodule about 1 cm in diameter slightly above, and lateral to her right areola. A specific dye is injected into the tissue around the tumor, and an incision is made to expose the lymphatic vessels draining the area, for the lymphatic vessels take up the dye - which is visible. The vessels can be then traced to expose the lymph nodes receiving the lymph from the lesion. Usually, one should expect that the first nodes encountered would be the ... A. B. C. D. E. anterior axillary (anterior pectoral) nodes. Rotter's nodes. parasternal nodes alongside the internal thoracic artery and vein, just lateral to, and deep to the area of the sternum. central axillary nodes. the apical, or infraclavicular nodes.

46.

When the mother heard her four year old son choking and coughing, and then found him in the kitchen struggling to breathe, she quickly observed the open bag of peanuts on the kitchen table. Because she could observe his frightened efforts at breathing she knew, as a former student at Ross Medical School, that his airway was not completely blocked. She quickly found transport to the Emergency Department in the hospital where she was performing her clinical rotations. She, correctly, informed the Attending Physician that her son had aspirated a peanut, and that it would be MOST LIKELY found within the A. B. C. D. E. right primary bronchus left primary bronchus trachea oral cavity lower part of the esophagus

47.

The patient had been diagnosed earlier in the week with Guillain-Barre disease. He was now in extreme respiratory distress. His thoracic wall contracted and relaxed violently, but there was little movement of his abdominal wall. The degenerative disease had obviously affected the muscle that is most responsible for increasing the longitudinal dimensions of the thoracic cavity (and pleural cavities). It should be obvious, therefore, that he was suffering from ... A. B. C. D. E. paralysis of his intercostal muscles, and loss of the "bucket-handle movement" of his ribs. generalized intercostal nerve paralysis that caused the loss of the "pumphandle movment" of his ribs. paralysis of his medial and lateral pectoral nerves, interrupting the function of his pectoralis major muscles, important accessory respiratory muscles. paralysis of the sternocleidomastoid muscles. degeneration of the myelin of his phrenic nerves.

48.

Two days after the patient's breathing had become assisted by mechanical ventilation, the patient with Guillain-Barre began experiencing severe cardiac arrhythmia, with perilously slow cardiac contractions, resulting in reduced cardiac output. This MOST LIKELY resulted from interruption of the contractile stimulus carried by the ... A. B. C. D. E. left vagus nerve right phrenic nerve preganglionic sympathetic fibers in upper thoracic spinal nerves. cardiac pain fibers carried by upper thoracic spinal nerves. ventral horn neurons of spinal cord levels T1 - T4.

49.

The young mother had contracted rubella during the first trimester of her pregnancy. You, as the Family Physician for her and her family, decide to tell her that the occurrence of this disease can result in certain congenital problems such as ... A. B. C. D. E. patent ductus arteriosus pulmonary stenosis problems with the eyes, such as cataracts deafness all of those congenital problems listed in this question.

50.

The newly-born baby was diagnosed as having the clinical and laboratory signs of CDH (congenital diaphragmatic hernia). In this condition, the infant would MOST LIKELY have ... A. B. C. D. E. a herniation of the left lung into the abdomen. an unusual, and potentially harmful opening in the right half of the diaphragm. a temporary, and obviously benign (harmless) defect of the diaphragm. pulmonary hypoplasia. increased abdominal breathing, with marked movement of the abdominal wall.

51.

It is medieval Transylvania and the peasants have finally revolted against the rule of the evil and tyrannical Count Vlad (the Impaler). Driving a wooden stake into the sleeping count's chest at the sternal angle (of Louis), they hope to pierce the left ventricle of his heart. They miss. Why? A. B. C. D. E. They were at the right level but their stake must have veered off at an angle The stake missed the heart to the left The stake missed the heart to the right The stake pierced the arch of the aorta The stake pierced the right subclavian artery

52.

The 22-year-old woman is to undergo bilateral (both sides) removal of the C6 T2 segments of her sympathetic chains, in an attempt to correct the excessive, chronic vasoconstriction in her upper limbs that was causing ischemia (loss of adequate blood supply) to her hands, especially in her fingers. You must caution her that this procedure will result in ... A. B. C. D. E. a slight drooping of her upper eyelids (partial ptosis). excessive facial sweating. an inability to smile symmetrically. dilation of her pupils, which will interfere with her night driving by automobile. loss of sensation from the upper and middle parts of her face.

53.

A 16-year-old male high school student, troubled with a bad case of acne, complains of severe headache and problems with movements of his eyes. It is noted that he also has a high fever and is sweating profusely. By the swelling of the tissues on the left side of his face from an infected pimple adjacent to the philtrum leading to the upper lip, you conclude, correctly, that an infection has spread retrograde from the area of the acute infection to the cavernous sinus by way of ... A. B. C. D. E. tributaries of the anterior division of the retromandibular vein into the internal jugular vein. ipsilateral (same side) facial vein into the ophthalmic veins. the left angular vein into the left superficial temporal vein. the common facial vein into the external jugular venous system. inferior labial vein into the deep facial venous system, and then into the posterior division of the retromandibular vein.

54.

The patient, Harry, was 58 years of age and obviously very overweight. He was admitted to the hospital complaining of severe, crushing, retrosternal pain which he said began while he was jogging in his neighborhood. The pain has not gone away after taking three nitroglycerine tablets, and the pain has lasted longer than when he had last experienced an anginal attack. His skin is clammy and unusually cool, his heartrate is 120 per minute. Laboratory results show that his mitral valve is not functioning properly because of ischemia of the posterior aspect of his left ventricle and his left posterior papillary muscle. From your previous knowledge of the patient's coronary arterial pattern as being "left coronary dominant," you make the correct guess that ... A. B. C. D. E. the left coronary circumflex artery is blocked by disease (fatty and calcified atherosclerotic plaque). the right coronary artery supplies the posterior one-third of the IVS. the LAD is almost completely closed, probably just beyond the origin of the first major branch to the IVS. the conus artery is totally occluded. the moderator band of the left ventricle has become ischemic.

55.

TW, a 42-year-old male, is obviously anxious and complains of chest pain. He has difficulty breathing unless he is sitting up (orthopnea), and coughs occasionally, bringing up small droplets of blood in his sputum. He says that he was diagnosed in his earlier years as having rheumatic fever. He states further that he has been experiencing difficulty swallowing food lately. Auscultation reveals a distinct cardiac murmur beneath the left nipple. A barium swallow reveals that the esophagus is significantly compressed and pushed posteriorly. Echocardiography and ECG confirm your initial diagnosis that the patient has A. B. C. D. E. regurgitation through the tricuspid valve. stenosis of the pulmonary valve. mitral valve disease, with regurgitation interventricular septal defect aortic coarctation

56.

The infant was brought to the "well baby" clinic for a routine checkup. The mother was a 46-year-old woman who was significantly overweight. The baby appears normal in almost all respects, except that she has a dimpling of the skin in the lower spine, with a tuft of hair growing from the area of the dimple. The mother has observed the peculiar, tiny patch of hair and expresses concern about it. You reassure the mother that this sort of thing can be seen in perhaps 25% of live births, and that it is nothing more than an example of ... A. B. C. D. E. meningomyelocele meningocele spina bifida occulta spina bifida cystica anencephaly

57.

A homeobox gene mutation occurs which affects the migration of neural crest cells from cervical segments into pharyngeal arches 3 through 6 of a developing male embryo. Soon after the infant is born he is diagnosed with DiGeorge Syndrome, with associated signs and symptoms of hypocalcemic seizures, thymic dysplasia, and widely spaced eyes (hypertelorism). The cardiac defect MOST LIKELY to be present in this patient is A. B. C. D. E. interatrial septal defect patent ductus arteriosus interventricular septal defect pulmonary artery stenosis persistent truncus arteriosus

58.

Upon histologic examination by the pathologist, it was noted that the portion of the cardiac conduction system that penetrates (passes through) the right fibrous trigone had become interrupted by ingrowth of collagenous tissue, leading to the arrythmia and, finally, the death of the patient. The specific part of the conduction system which was interrupted was the ... A. B. C. D. E. atrioventricular bundle of His atrioventricular node posterior internodal pathway right bundle branch left bundle branch

59.

The parasympathetic portion of the Autonomic Nervous System acts to maintain homeostasis, insuring the normal functions of processes vital to life. With regard to this system, it is well known that ... A. B. parasympathetic stimulation of the heart results in a negative chronotropic effect in cardiac activity. the parasympathetic supply to the vascular smooth muscle and sweat glands of the upper limbs passes into the limb in the adventitia of the axillary artery.

C. D. E. 60.

the chemical mediator released by postganglionic parasympathetic neurons at the target tissues is most commonly norepinephrine. The preganglionic parasympathetic neurons for the heart, lungs and esophagus have their cell bodies in the intermediolateral cell column of upper thoracic segments of the spinal cord. Postganglionic parasympathetic fibers usually travel through the gray communicating rami from the ganglionated chain.

At a three-month well-baby check-up, a pediatrician discovers that a female infant born with Turner's Syndrome (45,XO) has a slightly elevated systolic blood pressure. MRI imaging reveals that the primary cause of the hypertension is narrowing of the aortic lumen (coarctation). Collateral circulation through internal thoracic, anterior spinal, and suprascapular arteries has developed, rendering the hypertension lower than it would have been otherwise. Abnormal development of what structure has caused the coarctation in this patient? A. B. C. D. E. Left fourth aortic arch distal to the seventh intersegmental artery Right fourth aortic arch distal to the ligamentum arteriosum Left third aortic arch proximal to the seventh intersegmental artery Right first aortic arch proximal to the ligamentum arteriosum Left first aortic arch proximal to the ligamentum arteriosum

31.

A 60-year-old woman presents with tic douloureux which is characterized by excruciating facial pain over the distribution of the fifth cranial nerve. This involves sensory neurons whose cell bodies are located in the: A. B. C. D. E. Submandibular ganglion Trigeminal ganglion Geniculate ganglion Otic ganglion Nodose ganglion

32.

A 45-year-old man presented to his physician with facial paralysis and increased sensitivity to sound. Examination revealed a lesion of the facial nerve in the facial canal. This lesion is also LIKELY to result in: A. B. C. D. E. Problems with swallowing Loss of taste from anterior two-thirds of tongue Dizziness An atrophied tongue Loss of secretion from the parotid gland

33.

An 11-year-old boy who fell off his bike was taken to the pediatrician after developing a black eye. X-ray reveals that he has an epicranial hematoma which is located: A. Between the dense connective tissue layer and the epicranial aponeurosis B. In the fourth or loose connective tissue layer of the scalp C. Between the skull and the dura D. In the dense connective tissue layer of the scalp E. Between dura and subarachnoid meningeal layers

34.

A fracture of the foramen ovale is MOST LIKELY to result in: A. B. C. D. E. Auditory deficit Facial paralysis on affected side Inability to shrug the shoulder on affected side A dry eye on affected side Unilateral masticatory paralysis

35.

A 45-year-old male receives a stab wound to the left posterior triangle of his neck. This patient is MOST LIKELY to experience: A. B. C. D. E. Inability to shrug his left shoulder Deviation of his tongue to the left side Drooping of the left eyelid Bleeding from the left internal jugular vein Damage to his vagus nerve

36.

A 47-year-old man with angina pectoris was revealed by a coronary angiogram to have a blockage of his left coronary artery, proximal to its bifurcation into its two main branches. An elective coronary bypass procedure is carried out. While grafting a vessel onto the anterior interventricular artery (L.A.D), what accompanying vessel must be avoided? A. B. C. D. E. Small cardiac vein Anterior cardiac vein Middle cardiac vein Great cardiac vein Least cardiac vein

37.

A 10-year-old female with a history of allergies is taken to the pediatrician following an asthmatic attack. On physical examination, she has difficulty breathing, coughs and expels mucous. Bronchial smooth muscle (bronchospasm) and glands (mucous production) receive motor innervation from: A. B. C. D. E. Thoracic splanchnic nerves Sympathetic nerves Intercostal nerves The vagus nerve Cord levels, T1 to T 4

38.

A 38-year-old diabetic female is diagnosed with right-sided Bell's palsy, the result of injury to the facial nerve. Which of the following is MOST LIKELY to be seen in a patient with Bell's palsy? A. B. C. D. E. Loss of pain and temperature sense on the affected side of the face Inability to blink the right eye Paralysis of tongue muscles Inability to chew Inability to protrude the jaw

39.

A 16-year-old high school student with a bad case of acne complains of severe headache, high fever and problems with eye movements. Apparently, an infection had spread from a pimple on his face into the cavernous sinuses within his cranium through the: A. B. C. D. E. Angular and deep facial veins Facial and superior ophthalmic veins Retromandibular and internal jugular veins Facial and internal carotid arteries Middle meningeal and maxillary veins

40.

A patient is unable to laterally rotate her right upper limb against resistance. Which of the following are LIKELY reasons? A. Damage to rhomboids

B. C. D. E. 41.

Damage to rotator cuff muscles Damage to axillary nerve Damage to anterior and medial fibers of deltoid Damage to pectoralis major

Following a wedge fracture of the vertebral column a person complains of lack of sensation around the umbilicus. Which vertebra was MOST LIKELY involved? A. B. C. D. E. 10 Th. 12 Th. 2 Lumbar 6 Th. 8 Th.

42.

A penetrating knife wound in the 4th left intercostal space, just lateral to the sternum is MOST LIKELY to involve which chamber of the heart? A. B. C. D. E. Left ventricle Right ventricle Right auricle Right atrium Left atrium

43.

Which of the following structures are NOT involved with the triangle of auscultation? A. B. C. D. E. Lateral border of trapezius Teres minor Rhomboid major Inferior angle of scapula Supero-medial border of latissimus dorsi

44.

Which of the following veins DO NOT empty into the right atrium of the heart? A. B. C. D. E. Coronary sinus Inferior vena cava Anterior cardiac vein Superior vena cava Pulmonary veins

45.

Which of the following structures are NOT found in the posterior mediastinum? A. B. C. D. E. Azygous vein Sympathetic trunks Thoracic aorta Esophagus Trachea

46.

Which of the following statements about the heart is TRUE? A. B. C. D. E. Pulmonary valve is auscultated in 2nd right inter costal space Mitral valve is auscultated in right Sth intercostals space Chordae-tendinae attach papillary muscles to atrioventricular valves Wall of right ventricle is much thicker than that of left Base of heart is formed mainly by right atrium

47.

The apex of the heart is normally located in the: A. B. C. D. E. Mid clavicular line, 5th right intercostal space Mid clavicular line, 4th left intercostal space Mid clavicular line, 2nd right intercostal space Mid clavicular line, 5th left intercostal space Right border of sternum, near 5th costal cartilage

48.

Which of the following correlations is INCORRECT? A. B. C. D. E. GVE --- autonomic motor fibers Parasympathetic fibers --- to smooth muscles of blood vessels Efferent nerve fibers --- motor Dorsal root ganglion --- sensory cell bodies Sympathetic --- thoraco-lumbar outflow

49.

Which of the following correlations is INCORRECT? A. B. C. D. E. Latissimus dorsi --- medial rotation of humerus Occipital artery --- external carotid artery Tendons --- attach muscles to bone Adduction --- move limb to the body Greater occipital nerve --- dorsal primary rami of Cl

50.

In a patient with flail chest affecting ribs 5, 6, and 7 in the mid-clavicular line, there is a detached section of thoracic wall encircled by breaks in the ribs. During inspiration, in which direction does the detached section move? A. B. C. D. E. Superiorly, opposite to the diaphragm Inward, pulled in by negative pressure in the pleural space Outward, pushed out by positive pressure in the pleural space Inferiorly, along with the diaphragm There will be no movement

51.

A 27-year-old nulliparous woman who is five months pregnant tells her gynecologist that she thinks she has a lump in her right breast. On examination there is a slightly tender mass that extends from the upper right quadrant of the breast into the axilla. A mammogram, however, shows no evidence of a tumor or fibrosis in the breast. The "lump" that this patient felt was MOST LIKELY A. the axillary tail of Spence

B. C. D. E. 52.

suspensory ligaments of Cooper the anterior pectoral lymph nodes the axillary lymph nodes lymphatic vessels

A 10-year-old boy was lying on his back, throwing peanuts into the air and trying to catch them in his mouth, when a peanut went down his trachea and lodged in his lung. Which bronchopulmonary segment did the peanut MOST LIKELY enter? A. B. C. D. E. Right inferior superior Left apicoposterior Right apical Left inferior lingular Left anteromedial basal

53.

A 22-year-old woman who is eight months pregnant complains to her physician that her abdomen is becoming so large that it is painful. Upon investigation, polyhydramnios is diagnosed and amniotic fluid is removed, relieving the patient's discomfort. One month later, the woman gives birth to a cyanotic infant, a male with a hypoplastic thumb. In cutting the umbilical cord, the obstetrician also notices that there is a single umbilical artery. During his first feeding, milk spills from the neonate's mouth and then he violently coughs and gags. An acyanotic cardiac defect is also present. This defect is MOST LIKELY: A. B. C. D. E. Ventricular septal defect Common truncus arteriosus Transposition of the great vessels Tetralogy of Fallot Congenital absence of the interventricular septum (3 chambered heart)

54. A 63-year-old man who undergoes an arteriographic examination preparatory to coronary artery surgery is found to have a retroesophageal right subclavian artery which comes off the aorta distal to the origin of the left subclavian artery. This anatomical abnormality, of potential clinical significance if it impinges on the esophagus, can be explained as A. B. C. D. E. 55. an unusually highly placed left lateral thoracic artery a genetically mediated duplication of the left subclavian artery an aberrant left highest intercostal artery a remnant of the embryonic right 7th intersegmental artery a remnant of the embryonic right fourth aortic arch

The patient whose radiograph appears below shows a depressed diaphragm with unusually radiolucent and hyper-expanded lungs. This patient MOST LIKELY has

A. B. C. D. E. 56.

pneumothorax bronchopulmonary carcinoma pneumonia tuberculosis emphysema

Which of the following malformations is the basic defect in Tetralogy of Fallot? A. B. C. D. E. Pulmonary stenosis Coarctation of the aorta Patent ductus ateriosus Hypertrophy of the right ventricle Interventricular septal defect

57.

A 14-year-old girl presents to the ER with pyrexia, stiff neck and a rash. You suspect meningitis and decide to perform a lumbar puncture. Your needle will pass through the following structures in which CORRECT order? A. B. C. D. E. Skin, ligamentum flavum, supraspinous ligament, epidural space, dura mater, arachnoid mater Skin, supraspinous ligament, ligamentum flavum, epidural space, dura mater, arachnoid mater Skin, supraspinous ligament, ligamentum flavum, arachnoid mater, epidural space, dura mater Skin, ligamentum flavum, supraspinous ligament, arachnoid mater, epidural space, dura mater Skin, dura mater, arachnoid mater, ligamentum flavum, epidural space, supraspinous ligament

58.

Prostate cancer often spreads to the vertebral column via the internal vertebral venous plexus, where is this plexus found? A. B. Epidural space Subarachnoid space

C. D. E. 59.

Subcutaneous tissue Subdural space Foramen transversarium

When performing epidural anesthesia on adults, the following landmarks are used to ensure the spinal cord is not damaged on inserting your needle: A. B. C. D. E. The umbilicus The 12th thoracic vertebra The xiphisternum A horizontal line joining the highest points of the iliac crests The subcostal margin

60.

The cephalic veins can sometimes be used for emergency venous access. It lies in the ______________ and drains into the ________________. A. B. C. D. E. Deltopectoral groove; axillary artery Suboccipital triangle; brachial plexus Subcostal groove; axillary vein Costodiaphragmatic recess; brachial artery Deltopectoral groove, axillary vein

31.

Which of the following is CORRECT? A. B. C. D. E. The right primary bronchus is longer than the left primary bronchus The left primary bronchus is wider than the right primary bronchus The left primary bronchus divides into three secondary bronchi Aspirated (inhaled) foreign bodies pass more often into the left primary bronchus than into the right primary bronchus The right primary bronchus is more vertical in orientation than the left primary bronchus

32.

Which of the following is NOT characteristic of the third cervical vertebra? A. B. C. D. E. A demifacet on the body A pedicle A bifid spinous process A foramen in the transverse process Attachment of its spinous process to the nuchal ligament

33.

Which of the following is NOT characteristic of the right atrium or right auricle? A. B. C. D. E. Crista terminalis Ostium of the coronary venous sinus Trabeculae carneae Ostia (openings) for the anterior cardiac veins Valve of the inferior vena cava

34.

Which of the following statements is INCORRECT? A. B. C. T8 D. E. During a pleural tap, a needle is inserted close to the superior border of lower rib in the intercostal space to avoid the intercostal nerves Supernumerary nipples can occur in males or females The inferior vena cava passes through the diaphragm at vertebral level Cutting the medial pectoral nerve at its origin would cause total paralysis of the clavicular head of the pectoralis major muscle The nipple receives its sympathetic and sensory supply from the lateral cutaneous branch of the fourth thoracic nerve

35.

Damage to the facial nerve at the stylomastoid foramen would MOST LIKELY cause A. B. C. D. E. inability to open the eyelids weakness in chewing on the side of the nerve damage reduction in sensation from the anterior part of the scalp inability to smile symmetrically inability to protrude the jaw

36.

A 20-year-old man presents to the doctor with severe headache, high fever and a stiff neck. A lumbar puncture is performed to determine whether or not he

has meningitis (infection of the meninges). Where is the BEST site to perform this puncture? A. B. C. D. E. 37. At the level of a line,drawn between the iliac crests Between vertebra Ll and L2 At vertebra L2 Between vertebrae L2 and L3 At vertebra S2

The skin of the face receives its sensory supply primarily from branches of A. B. C. D. E. ventral primary rami of Cl and C2 the superior, middle and inferior auricular nerves the facial nerve the cervical plexus the trigeminal nerve

38. The cephalic vein can sometimes be used for emergency venous access. It lies in the__________ and drains into the _________ A. B. C. D. E. Deltopectoral groove; subclavian vein Deltopectoral groove; external jugular vein Intertubercular groove; axillary vein Bicipital groove; brachial vein Deltopectoral groove; axillary vein

39. At birth, the lower end of the conus medullaris is normally found at vertebral level A. B. C. D. E. 40. S-2 T-12 L-1 L-3 COC-2

Select the INCORRECT statement regarding the facial blood vessels: A. B. C. D. E. The retromandibular vein is formed by the superficial temporal and maxillary veins The pulse felt over the zygomatic arch, in front of the ear, is that of the superficial temporal and maxillary veins The facial vein most commonly drains into the external jugular vein The facial artery arises from the external carotid Infected facial venous blood can get into intracranial veins (the cavernous sinus, for instance) by way of the ophthalmic vein

41.

Which of the following statements is INCORRECT? A. Spina bifida most commonly occurs at the cervicothoracic junction

B. C. D. E. 42.

Spina bifida occulta is the least severe variation of spina bifida Spina bifida results from failure of vertebral arches to fuse Spina bifida cystica involves the meninges Spina bifida with myeloschisis is the most severe variation of the defect

Select the TRUE statement regarding the neck: A. B. C. D. E. The axillary sheath that surrounds the brachial plexus is derived from the prevertebral fascia of the neck Torticollis results from injury to the facial nerve The platysma is ensheathed (covered) by the cervical investing fascia If a sternocleidomastoid muscle is paralyzed, there will be weakness in turning the head to the side of the paralyzed muscle The first rib provides the base for the posterior cervical triangle

43. The MOST IMPORTANT factor in increase of intrathoracic volume in inspiration is A. B. C. D. E. 44. the "pumphandle movement" of the ribs the contraction of intercostal muscle the "buckethandle movement" of the ribs contraction of the diaphragm the ascent of the sternum by contraction of the sternocleidomastoid

The pulmonary lymphatic drainage is the most important channel in the spread of lung cancer. Which of the following nodes DOES NOT normally receive lymph from the lung? A. B. C. D. E. Tracheobrochial nodes Axillary nodes Hilar nodes Bronchomediastinal nodes Tracheal nodes

45.

Regarding the parotid salivary gland, which of the following statements is INCORRECT? A. B. C. D. E. The posterior surface of the parotid gland is in contact with the sternocleidomastoid muscle The parotid duct empties opposite the second upper molar tooth This is the gland that is infected in mumps The parotid gland is invested with dense connective tissue so that any swelling of the gland results in pain A lesion of the facial nerve at the stylomastoid foramen results in interruption of the nerve supply to the parotid and secretion is reduced

46.

Which of the following statements is NOT TRUE? A. B. C. D. E. The mandibular division of the trigeminal nerve is the only division which supplies skeletal muscles The facial nerve does not supply sensation to the cheek The platysma muscle receives its motor supply from neurons derived from C2, C3 which are carried by the transverse cervical and greater auricular nerves The buccal branch of the trigeminal nerve does not provide motor supply to the buccinator muscle Bell's palsy is not usually associated with physical pain

47.

A bullet enters the right lateral chest wall at level of the 8th intercostal space. It passes through all layers from skin through and including the parietal pleura. From the following list choose the position where one might logically expect to find the bullet: A. B. C. D. E. Endothoracic fascia Pericardial cavity middle mediastinum Pleural cavity middle lobe of right lung

48.

The sternal angle is an important landmark. Which of the following is NOT related to it? A. B. C. D. E. Sternoclavicular joint Aortic arch Bifurcation of trachea 2nd costal cartilage Manubriosternal joint

49.

The smooth part of the right atrium is mainly derived from the A. B. C. D. E. truncus arteriosus right pulmonary vein bulbus cordis primitive pulmonary vein sinus venosus

50.

Which of the following statements is INCORRECT? A.

The intercostobrachial nerve is usually related intimately with the axillary tail of the breast B. The serratus posterior superior has nothing to do with movement of the scapula C. Trapezius paralysis will make it difficult to elevate (abduct) the limb above the level of the shoulder D. E. If the long thoracic nerve were cut, there would be weakness in retracting the scapula Paralysis of the thoracodorsal nerve results in weakness in extension of the arm

51.

Congenital heart disease is the MOST COMMON cardiac problem in childhood and MOST FREQUENTLY results from A. B. C. D. E. fetal distress maternal medications mutant genes rubella virus genetic and environmental factors

52.

Tetralogy of Fallot is defined as a complex of cardiac malformations which includes A. B. C. D. E. origin of the aorta totally from the right ventricle instead of the left ventricle a severe narrowing,of the proximal part of the pulmonary artery a total absence of the interatrial septum due to resorption of septum secundum an early closure of the ducts arteriosus before birth a right-sided aortic arch

53.

What nerve DOES the superior laryngeal artery accompany into the larynx? A. B. C. D. E. Vagus Descendens hypoglossal of ansa cervicalis The nerve to the cricothyroid muscle Recurrent laryngeal Internal laryngeal

54.

The costodiaphragmatic recess: A. B. C. D. E. Contains the cupola Is found between the heart and lungs Is a recess associated with visceral pleura Is associated with the pulmonary ligament Is a recess associated with parietal pleura

55.

Which of the following is TRUE? A. B. C. D. E. The subarachnoid space stops inferiorly at about vertebral level L2 The lateral border of the "triangle of auscultation" of the back is provided by the levator scapulae muscle The sensory supply to the skin of the lower eyelid is supplied by a branch of the mandibular division of the trigeminal nerve The artery to the sinoatrial node most commonly arises from the left coronary artery The sympathetic chain lies posterior to, not within, the carotid sheath

56.

The lymph nodes to which lymph passes FIRST from the areola and nipple are the

A. B. C. D. E. 57.

sentinel (scalene) nodes apical nodes anterior pectoral (anterior axillary) nodes central axillary nodes, interpectoral nodes (of Rotter)

Which of the following statements is FALSE? A. B. C. D. E. The internal vertebral venous plexus, also known as (Batson's veins), lies within the epidural space The nucleus pulposus is derived from the notochord The spinal cord is separated from the vertebral bodies by dura and the posterior longitudinal ligament The cerebrospinal fluid is found primarily within the subdural space A herniated disk between L4 and L5 would most likely compress spinal nerve L5

58.

Which of the following statements is INCORRECT? A. B. C. D. E. All spinal nerves contain the same types of functional components A typical spinal nerve contains four functional components General somatic efferent fibers supply striated voluntary muscles The 1st cervical nerve is generally considered to be a purely motor nerve General visceral efferent fibers control vasoconstriction and dilation of blood vessels

59.

The MOST COMMON type of defect of the cardiac septa is: A. B. C. D. E. Primum type ASD Membranous type VSD Secundum type ASD Muscular type VSD Common atrium

60.

Which of the following statement is INCORRECT? A. B. C. D. E. The middle meningeal artery lies in the subarachnoid space The middle meningeal artery is often encircled by the auriculotemporal nerve The middle meningeal artery enters the skull via the foramen spinosum The pterion of the - skull is an important landmark for the middle meningeal artery The middle meningeal is a branch of the maxillary artery

36.

A 65-year-old man is involved in a road traffic accident. A CT scan shows he sustained an isolated fracture of the posterior cranial fossa involving the jugular foramen. Which of the following signs and symptoms is he MOST LIKELY to be suffering from? A. B. C. D. E. Loss of hearing Deviation of his tongue on protrusion Loss of smell Exopthalmos A sagging shoulder

37.

Concerning the meninges: A. B. C. D. E. They are sensitive to pain Cerebral arteries and veins lie in the subarachnoid space A ruptured meningeal artery will cause a subdural haemorrhage The dural venous sinuses lie between the dura mater and the arachnoid mater They receive nerve supply from the cranial part of the accessory nerve

38.

A 14-year-old girl with severe facial acne is brought unconscious to the ER. She is diagnosed with a cavernous venous thrombosis. Through which of the following vessels did the infection spread? A. B. C. D. E. Ophthalmic vein Internal carotid artery Facial artery Retromandibular vein Superior sagittal sinus

39.

A 26-year-old lady has a little too much to drink and falls. She sustains a scalp laceration that has gaped. Which layer of the scalp causes gaping when breached? A. B. C. D. E. Aponeurosis Loose areolar tissue Skin Connective tissue Periosteum

40.

A 35-year-old man is stabbed through the 9th right intercostal space in the mid-axillary line during mid-inspiration. Which structures are likely to have been pierced? A. B. C. D. E. Skin, intercostal muscles, parietal pleura, visceral pleura Skin, intercostal muscles, parietal pleura Skin, parietal pleura, visceral pleural, lung Intercostal muscles, parietal pleura, lung Skin, intercostal muscles, visceral pleura

41.

A 5-year-old boy inhales a small marble. In which segment is it MOST LIKELY to lodge?

A. B. C. D. E.

Left superior lingual Right apical Right medial basal Left apical Left medial basal

42. A 50-year-old man presents to the ER with severe chest pain and a CT scan is performed. The structure marked X is:

A. B. C. D. E. 43.

Esophageal cancer Dilated thoracic duct A false lumen in a dissecting thoracic aneurysm Superior Vena Cava aneurysm Azygos Vein

Which of the following statements concerning the thoracic duct is INCORRECT? A. B. C. D. E. Ascends through the aortic opening of the diaphragm Its enlarged lower end is called the cisterna chyli It drains all lymphatics below the diaphragm Normally drains into the origin of the left brachiocephalic vein Lies in front of the esophagus

44.

When performing epidural anesthesia on adults, the following landmarks are used to ensure the spinal cord is NOT damaged on inserting your needle: A. B. C. D. E. The umbilicus The 12th Thoracic vertebra Spine of 1st sacral vertebra A horizontal line joining the highest points of the iliac crests The subcostal margin

45.

In the examination and investigation of a patient with longstanding mitral stenosis, which of the following statements is CORRECT? A. B. C. D. E. The mitral valve is best heard at the level of 5th left intercostal space, at the midclavicular line The mitral valve is found between the right atrium and right ventricle The left ventricle will hypertrophy A barium swallow will show anterior displacement of the esophagus The mitral valve has 3 cusps

46.

The cephalic vein can sometimes be used for emergency venous access. It lies in the and drains into the A. B. C. D. E. suboccipital triangle; vertebral vein subcostal groove; axillary vein deltopectoral groove; brachial vein deltopectoral groove; axillary artery deltopectoral groove; axillary vein

47.

A 14-year-old girl presents to the ER with pyrexia, stiff neck and a rash. You suspect meningitis and decide to perform a lumbar puncture. Your needle will pass through the following structures in which CORRECT order: A. B. C. D. E. Skin, ligamentum flavum, supraspinous ligament, arachnoid mater, epidural space, dura mater Skin, ligamentum flavum, supraspinous ligament, epidural space, dura mater, arachnoid mater Skin, supraspinous ligament, ligamentum flavum, epidural space, dura mater, arachnoid mater Skin, supraspinous ligament, ligament flavum, arachnoid mater, epidural space, dura mater Skin, dura mater, arachnoid mater, ligamentum flavum, epidural space, supraspinous ligament

48.

Prostate cancer often spreads to the vertebral column via the internal vertebral venous plexus. Where is this plexus found? A. B. C. D. E. Subdural space Epidural space Subarachnoid space Skin Foramen transversarium

49. Which of the following statements are NOT TRUE of an intervertebral disc? A. B. C. D. E. The disc influences the development of the curvatures of the vertebral column During aging the nucleus pulposus can be replaced by fibrocartilage The discs are wider in the lumbar region The atlanto-axial joint has no disc The nucleus pulposus is most likely to herniate in an antero-lateral direction

50. Which of the following vertebrae is missing a body? A. B. C. D. E. Atlas Thoracic Lumbar Axis Sacral

51. Which of the following statements is INCORRECT regarding the sternal angle of Louis? A. B. C. D. E. It is where the 3rd pair of costal cartilages attach to the sternum It is used to indicate where the trachea bifurcates It is located at the vertebral level of T4/T5 It is used to indicate where the arch of the aorta begins It is used to indicate where the arch of the aorta ends

52. Which of the following statements is INCORRECT? A. B. C. D. E. Involvement of level one axillary lymph node carries the worst prognosis Polythelia is a condition in which there are accessory nipples The majority of lymph from the breast goes to the axillary lymph nodes The pectoralis minor muscles is used to divide axillary lymph nodes into 3 levels Skin dimpling is a sign of breast cancer

53. Which of the following statements about the cubital fossa is CORRECT? A. B. C. D. E. Contains the u1nar nerve The cephalic vein overlies the bicipital aponeuroses The floor is formed by the head of flexor digitorum superficialis The median nerve lies lateral to the brachial artery The brachial artery lies immediately medial to tendon of the biceps

54. Which of the following statements is TRUE regarding the axillary nerve? A. B. C. The axillary nerve may be compressed or injured when the glenohumeral joint dislocates The axillary nerve is accompanied by the profunda brachii artery The axillary nerve innervates the deltoid and teres major muscles

D. E.

The axillary nerve is a branch of the medial cord of the brachial plexus The axillary nerve is seen in the triangular interval of the humerus

55. In a posterior surgical approach to the glenohumeral joint, an osteotomy of the greater tubercle is performed (separation from the humerus and reflection with its musculotendinous attachments intact). Which of the following rotator cuff muscles would NOT be reflected with this operation? A. B. C. D. E. 56. Teres minor Supraspinatus Subscapularis Infra spinatus B&D

A 67-year-old woman slipped on a scatter rug and fell with her right arm extended in an attempt to ease the impact of the fall. She experienced immediate severe pain in the region of the right collarbone and in the right wrist. Painful movement of the right arm was minimized by holding the arm close to the body and by supporting the elbow with the left hand. The clavicle is commonly fractured due MOSTLY to which of the following factors? A. B. C. D. E. Subcutaneous location It is the only strut that transmits force from arm to trunk Last bone to ossify First bone to ossify It is weaker in adults than in children

57.

The humerus, scapula and lateral fragment of the clavicle drop due to gravity, with the lateral fragment pulled medially (subluxed), underriding the medial fragment. This is caused by contraction of which of the following muscles? A. B. C. D. E. Pectoralis minor muscle Supraspinatus muscle Deltoid muscle Pectoralis major muscle Latissimus dorsi muscle

58.

Internal bleeding can be a complication if the subluxed bone fragment tears a vessel and punctures the pleura. Which of the following vascular structures is particularly vulnerable in a clavicular fracture? A. B. C. D. E. Subclavian artery Axillary artery Brachiocephalic artery Lateral thoracic artery Thoracoacromial trunk

59.

Which of the following is NOT characteristic of the right atrium or right auricle? A. B. C. D. E. Trabeculae carneae Ostia (openings) for the anterior cardiac veins Crista terminalis Ostium of the coronary venous sinus Valve of the inferior vena cava

60.

Which of the following statement is INCORRECT? A. B. C. D. E. Spina bifida most commonly occurs at the cervicothoracic junction Spina bifida occulta is the least severe variation of spina bifida Spina bifida results from failure of vertebral arches to fuse Spina bifida cystica involves the meninges Spina bifida with myeloschisis is the most severe variation of the defect

61.

The MOST IMPORTANT factor in increase of intrathoracic volume in quiet inspiration is A. B. C. D. E. the "buckethandle movement" of the ribs contraction of the diaphragm the "pumphandle movement" of the ribs the contraction of intercostal muscle the ascent of the sternum by contraction of the sternocleidomastoid

62.

Occlusion (blockage) of the left coronary artery is MOST LIKELY to stop blood from flowing into the: A. B. C. D. E. Right marginal artery Posterior interventricular artery Anterior interventricular artery Right coronary artery S.A. nodal artery

63.

In angina pectoris, the pain radiating down the left arm is associated with increased activity of sensory (afferent) fibers that travel along sympathetic pathways to reach the: A. B. C. D. E. Upper two lumbar spinal cord segments Brain stem Upper four or five cervical spinal cord segments Upper four or five thoracic spinal cord segments None of the choices are correct

64.

Which of the following group of nerves is intimately related to a portion of the humerus and is MOST LIKELY to be affected by fractures of the humerus? A. B. Axillary, median, musculocutaneous Axillary, musculocutaneous, radial

C. D. E. 65.

Axillary, median, ulnar Axillary, radial, ulnar Median, radial, ulnar

Inability to supinate the forearm could result from an injury to which of the following pairs of nerves? A. B. C. D. E. Musculocutaneous and median Axillary and radial Radial and musculocutaneous Suprascapular and axillary Median and ulnar

66.

A 50-year-old woman was recently diagnosed with carpel tunnel syndrome. Which of the following is LEAST LIKELY to be a predisposing factor in this condition? A. B. C. D. E. Tenosynovitis of long flexor tendons Compression of ulnar nerve Dislocated lunate Thickened flexor retinaculum Paresthesia or lack of sensation over thenar eminence

67.

A 21-year-old girl was thrown from her horse while attempting a difficult jump. She landed on her left shoulder and the left side of her head. It was noticed after a week of hospitalization that she kept her left arm medially rotated and close to her side with the forearm pronated. At physical examination, an area of anesthesia was found along the lateral side of the upper part of the arm. Which nerve(s) was (were) damaged during the accident? A. B. C. D. E. The middle trunk of the brachial plexus The lower trunk of the brachial plexus The axillary nerve The radial nerve Upper trunk of the brachial plexus

68.

The position adopted by the left arm in this patient can be explained by paralysis of which of the following groups of muscles? A. B. Triceps, anconeus, brachioradialis and supinator Flexor digitorium superficialis, flexor pollicis longus, flexor carpi radialis longus and flexor digitorium profundus C. Extensor carpi ulnaris, supinator, extensor indicis and extensor digit minimi D. Supraspinatus, infraspinatus, teres minor, deltoid and biceps brachii E. Pectoralis major, pectoralis minor, coracobrachialis and teres major

69.

A medical student noticed when he compressed both his radial artery and ulnar artery his hand goes pale, but when he compressed only one of them there is no change in color of the hand. Which structure is MOST LIKELY to contribute to this finding? A. B. C. D. E. Median artery Anterior interosseous artery Posterior interosseous artery Deep palmar arch Ulnar collateral artery

70.

A young mountain climber had fallen some distance before managing to grasp a safety line with one hand. The quick upward pull of the line on the weight of his body caused severe injury to the lower trunk of the brachial plexus. As a result of the lower trunk injury, one would CORRECTLY expect: A. B. C. D. E. loss Inability to flex the elbow and anesthesia on the lateral side of the ventral surface of the forearm Loss of function of the intrinsic muscles of the hand, with, anesthesia along the medial aspect of his forearm and hand Loss of extension of the elbow and anesthesia on the dorsum of his arm and forearm Paralysis of the deltoid and the "SIT" group of muscles and anesthesia over the proximal, lateral side of the arm Paralysis of the serratus anterior and scapular winging, without sensory

36.

The elderly fisherman was brought by his daughter to the clinic at Roseau. She states that he seems very confused lately; that his memory is not as sharp as she knows it usually to be, and that he doesn't move with his usual quickness and athleticism. She does not initially remember any injuries that he might have suffered. Your examination confirms the patient's lack of equilibrium and memory deficits. However, after you observe, and ask about a partially healed scalp laceration over the area of the lambda, she remembers that he complained about 10 days previously of having slipped and hitting his head on the mast of his boat. He said that he did not lose consciousness. Lumbar puncture reveals nothing; that is, there is no trace of blood in the CSF. You can, therefore, CORRECTLY conclude that ... A. B. C. D. E. The middle meningeal artery was torn in the unhappy meeting of the patient's head and the mast, resulting in an epidural hematoma. A cerebral vein was torn, resulting in slow accumulation of blood in the subdural space. When he slipped, the patient suffered a hemorrhage from rupture of a previously existing cerebral aneurysm, bleeding into the subarachnoid space, resulting in his diminished awareness and weakness. The fisherman is lucky to be alive, for he obviously suffered a skull fracture in the vicinity of the junction of the sphenoid, frontal, temporal and parietal bones. The fisherman and his daughter are simply scheming to collect insurance money for the fall and injury suffered from his clumsiness.

37.

The patient was a 22 YO female, a visitor to the Ross campus, who complains that she is having problems with her right eye. To examine the eye, you find it necessary to elevate her superior palpebra with your finger, because of her inability to raise it normally. Her right pupil is deviated to the right and slightly downward. Her complaint about your bright examination light seems due to the dilation of her right pupil. Cerebral imaging studies reveal that one of her cranial nerves is being compressed by an aneurysm of an adjacent artery, just above the tentorium cerebelli, adjacent to the tentorial notch. The neural structure which is being affected by the aneurysm is the ... A. B. C. D. E. right trochlear nerve right abducens nerve right oculomotor nerve right ophthalmic nerve optic chiasm

38.

The artery which might MOST reasonably cause the nerve compression described in the previous question would be the ... A. B. C. D. E. left superior cerebellar artery right posterior cerebral artery. basilar artery right posterior communicating artery right ophthalmic artery

39.

The 25 YO laborer is your subject for a physical examination in the ICM course, an examination which will determine your academic fate in the next semester. The presenting complaint is cranial nerve dysfunction. As the physical exam proceeds, you gently stroke his left limbus with a cotton swab and note, with some surprise, that there is an ipsilateral response, but no contralateral response. You confirm your initial diagnosis by stroking his right limbus and observing a blink response only of the left upper eyelid. With this information you can rightly conclude that he has paralysis of his A. B. C. D. E. right facial nerve left oculomotor nerve left facial nerve right ophthalmic nerve right optic nerve and right oculomotor nerve

40.

A 57-year-old woman who has smoked two packs of cigarettes a day for the past thirty years presents to your office with shortness of breath. She reports that she has difficulty in expelling air from her lungs, a classic sign of emphysema. On physical examination you notice that there is significant recruitment of accessory muscles of respiration while the patient is breathing. Contraction of which of the following muscles would be MOST SIGNIFICANT during the expiratory phase of respiration? A. B. C. D. E. Diaphragm Internal intercostal Scalenes Sternocleidomastoid Pectoralis minor

41.

An inexperienced emergency aid worker approaches the scene of an accident and finds a man unconscious and not breathing. He attempts a tracheostomy in order to place an oxygen tube in the patient's windpipe. However, blood begins to spurt briskly out of the deep midline incision that the technician has made in the patient's neck below the cricoid cartilage. What mistake has the technician made? A. B. C. D. E. Cut through the middle thyroid vein Forgotten that 10% of people have a thyroid ima artery Made the incision too low and cut the inferior thyroid artery Made the incision too high and cut the superior laryngeal artery Misidentified the thyrohyoid membrane

42.

Which of the following structures would be MOST LIKELY subject to congenital malformation of structures derived from the first branchial (pharyngeal) arch? A. B. C. D. E. styloid process buccinator stapes mandible upper portion of the hyoid bone

43.

She had suffered a severe knife wound to the upper lateral portion of her pectoral region, with entry of the knife at the deltopectoral groove. Pressure applied to the wound had prevented further profuse bleeding. In the Emergency Department, vascular clamps were applied to the axillary artery, proximal and distal to the site of injury which had occurred between the second and third parts of the axillary artery. The vascular surgeon knew that he would have adequate time to repair the wound of the artery because of the rich collateral pathway provided by the anastomoses between the ... A. B. C. D. E. suprascapular and circumflex scapular arteries supreme (superior) thoracic and thoracoacromial transverse cervical and suprascapular arteries posterior humeral circumflex and profunda brachii arteries lateral thoracic and suprascapular arteries

44.

A SO-year-old man is involved in an automobile accident and is brought into the Emergency Department with multiple injuries including trauma to the right side of the face and head. After his condition is stabilized a neurologic examination is performed. The patient complains of double vision. When you ask him to look downward and medially, you observe that the patient simply cannot turn the right pupil from an adducted position to a depressed position; that is, cannot look down and medially with that eye. What nerve(s) has (have) been injured? A. B. C. D. E. right oculomotor nerve right abducens nerve right trochlear nerve right abducens and trochlear nerves right oculomotor and trochlear nerves

45.

While on a fox hunt near Raleigh, North Carolina (where you are doing a 6 week rotation), a 21-year-old woman is thrown from her horse, which accidentally tramples her. She is kicked in the right side of the head at the pterion, and is dazed but not unconscious when her companions reach her. On the way back to the stable, however, she collapses and loses consciousness. When she is presented to you for examination, her right pupil is dilated in comparison with the left pupil. Her injury is MOST LIKELY A. B. C. massive intracerebral bleeding rupture of the internal carotid artery in the cavernous sinus tearing of the sigmoid sinus and internal jugular vein

D. E. 46.

a subdural hematoma epidural hematoma

The 105 YO female patient from Portsmouth, Dominica seems completely unaware of an ugly ulceration on the tip of her tongue. As you record this observation, you review the pathway of the pain fibers from the region, remembering that the cell bodies for pain, pressure and temperature from the anterior part of the tongue are located in the ... A. B. C. D. E. semilunar (trigeminal) ganglion spiral ganglion inferior vagal (nodose) ganglion inferior glossopharyngeal ganglion geniculate ganglion

47.

The 70 YO hypertensive patient had suffered numerous problems from her elevated blood pressure and vascular fragility. The most recent problem was a severe nosebleed from the anteroinferior portion of her nasal septum. Which of the following arteries would be LEAST LIKELY to be involved directly in such bleeding? A. B. C. D. E. greater palatine anterior ethmoidal posterior ethmoidal sphenopalatine superior labial

48.

The little girl was obviously small for her age and her eyes were so far apart that you include hypertelorism as one of your observations during the physical exam. In response to your cautious questioning, the mother tells you that the child suffers from recurring colds. Laboratory studies reveal that the little patient also has a condition of hypocalcemia, leading you to decide on a diagnosis of DiGeorge Syndrome. Which of the following might BEST explain the chronic colds the little girl is suffering? A. B. C. D. E. internal branchial fistula ectopic parathyroid glands hypoplastic thymus gland persistent thyroglossal duct cervical cysts

49.

The 30 YO male medical student enters the campus clinic following one of the Embryology lectures, seeking confirmation of his self-diagnosis. The student has a small, soft lump on the anterior border of his left sternocleidomastoid muscle and he has decided to ask the opinion of the campus physician about it. A mucoid secretion can be seen oozing from the soft mass of tissue. The clinician's classic training leads him to agree with the medical student that the proper diagnosis is ...

A. B. C. D. E. 50.

branchial or pharyngeal fistula aberrant thymic tissue persistent 2nd pharyngeal pouch inflamed and severely infected deep cervical lymph nodes ectopic palatine tonsil

The left side of the face of the 60 YO fruit salesman in the clinic appears to be strangely smooth, with absence of the nasolabial furrow and less wrinkling of the skin of his forehead than on the contralateral side. The corner of his mouth on the left side droops noticeably. He complains that he had to stuff a little piece of cloth into his left ear because sounds heard with that ear seemed so loud (hyperacusis). Physical examination reveals absence of the sense of taste on the left side of his tongue anteriorly. Function of his lacrimal gland on that side seems to be normal. With these facts in mind, you can conclude that the lesion of the involved nerve is probably located ... A. B. C. D. E. in the petrous part of the temporal bone, proximal to the location of the geniculate ganglioin at the stylomastoid foramen in the internal acoustic (auditory) meatus in the petrous part of the temporal bone, distal to the origin of the greater petrosal nerve within the parotid gland

51.

The knowledge that important nerves often accompany named arteries (so-called "neurovascular pairs") can be of great assistance to the clinician when looking for a particular nerve. An example of such an association is that of the ... A. B. C. D. E. angular artery and the buccal branch of the facial nerve the external laryngeal nerve with the superior laryngeal artery the inferior thyroid artery and the recurrent laryngeal nerve superficial temporal artery and the facial nerve the lingual artery and the buccal nerve

52.

Magnetic resonance imaging can provide excellent visualization of soft tissues of the body. With such imaging methodology, for instance, it could be demonstrated that one of the following DOES NOT pass through the superior orbital fissure: A. B. C. D. E. trochlear nerve abducens nerve ophthalmic artery oculomotor nerve superior ophthalmic vein

53.

A 14 YO female patient from Calibishe, Dominica is seen in the Portsmouth hospital by the small group of nervous 4th semester students (nervous, but highly self-confident). It is immediately apparent that the girl has a bad case of acne. She complains of severe headache, pain superficially and deeply in her

midface, and loss of all voluntary movement of her right eye. She has an elevated temperature. Physical and ophthalmoscopic examination reveal ipsilateral ptosis, a swollen palpebra and engorged retinal veins. Which of the following combinations of structures possibly involved in this case of coronary sinus thrombosis might BEST explain the ocular findings? A. B. C. D. E. 54. CN VII and CN VI CN III, CN IV and CN VI CNV1, CNV2 and CN III CN I, CN VI, CNV, and CNV2 CN III, CN IV, CN VI and CN V3

While treating young children from the local public school, you examine the swollen, and angry-red eardrum of Carolon, a 7 YO girl with a middle ear infection. Which of the following statements do you remember as being TRUE, as you review the clinical anatomy of the area? A. B. C. D. E. Umbo is a drug formulated from leaves of the dashish plant to be inserted into the ear to cure infections in the middle ear and the pharyngotympanic tube. The "cone of light" is due to reflection of the otoscope light toward the posterior inferior quadrant of the eardrum. The vagus nerve, auriculotemporal nerve and glossopharyngeal nerve are all involved in the sensory supply of the eardrum. An incision for drainage of a middle ear infection should be made in the upper, posterior quadrant of the tympanic membrane. The tympanic branch of the glossopharyngeal nerve is called "chorda tympani" because it crosses the upper inner aspect of the eardrum.

55.

The 72 YO woman had a lesion in the middle of her right vocal cord which biopsy revealed to be carcinoma. Further treatment would be dictated by the position of the tumor and by the location of the lymph nodes draining the area. The lymph nodes which receive lymph directly from this part of the vocal cord are the ... A. B. C. D. E. juguloomohyoid node sentinal node submental nodes jugulodigastric node None of the above is a correct response

56.

During the Saturday morning Morbidity and Mortality Conference in the Staten Island Hospital, the conclusion was offered that the patient under discussion had almost died as a result of asphyxiation (suffocation) because both recurrent laryngeal nerves had been injured during the removal of the patient's diseased thyroid glands. The young resident, a graduate of Ross Medical School, left no doubt in the minds of those attending the conference that one pair of laryngeal muscles would still be innervated after bilateral injuries to the recurrent laryngeal nerves. The continuing contraction of this muscle brought the vocal

cords together in the heavily sedated patient, thereby blocking the airway. The financial loss to the hospital in a lawsuit would have been great if she had not observed the problem and ordered respiratory intubation for the patient. The muscle that was still functioning was the ... A. B. C. D. E. 57. cricothyroid posterior cricoarytenoid arytenoideus lateral cricoarytenoid thyroarytenoid

The fourth semester student was asked by the examining clinician in the ICM program to check the functions of the masticatory muscles in an elderly woman from Portsmouth. After the student asked the woman to clench her jaws together tightly he could feel on palpation that there was a lack of symmetry in size and strength of the muscles from one side of the jaw to the other, as noted both by the lack of fullness under his examining fingertips and the unequal strength of the jaws as the patient closed her mouth against resistance. When the patient was asked to "stick out your chin," (in other words, protrude her mandible directly forward) the mandible deviated to the right. This would MOST LIKELY occur as a direct result of paralysis to the ... A. B. C. D. E. posterior segment of her left temporalis right medial pterygoid left masseter right lateral pterygoid left lateral pterygoid

58.

As you examine the baby who was delivered two weeks earlier at Princess Margaret Hospital (Roseau) you note that the left upper limb is rotated medially and that the infant is seemingly unable to move the limb at the shoulder. Finger movements of the ipsilateral limb indicate that distal muscles are functional. From this, you conclude CORRECTLY that an injury has occurred at birth to the. A. B. C. D. E. lower trunk of the brachial plexus axillary nerve upper trunk of the brachial plexus musculocutaneous nerve C7 ventral primary ramus

59.

The 10 YO boy had been teased cruelly by the other children in his class, the mother told you, because his head had been tilted to one side since birth. You respond by telling her that the lad has congenital torticollis and that it can be corrected rather easily by a surgical procedure. The condition resulted from fibrosis of the ... A. B. C. splenius musculature, unilaterally levator scapulae muscle sternocleidomastoid

D. E. 60.

omohyoid ipsilateral strap muscles

The second semester student had gone to Roseau because pain from a bad molar tooth was distracting him from his preparations for the second Mini exam at Ross. The dentist who treated the student injected a bolus of anesthetic into the region just anterior to the mandibular lingula. In a few minutes, the student observed that he had lost all sensations on the anterior parts of the tongue ipsilateral to the injection. He realized that the loss of tongue sensation was due to the fact that the anesthetic had directly affected the ... A. inferior alveolar nerve B. auriculotemporal nerve C. lingual nerve D. buccal nerve chorda tympani nerve Unilateral clefts of the upper lip are common, occurring about once in 1000 births. The underlying cause for this congenital defect is known to be the lack of fusion.... A. B. C. D. E. between palatine shelves between the primary and secondary palate and distorsion of the lateral palatine processes, preventing their migration of the maxillary processes between maxillary and mandibular processes between a maxillary prominence and the medial nasal prominence

E. 61.

62.

The recurring infections in the young boy seemed to indicate a need for removal of his palatine tonsils. These lymphoid organs are found in the... A. B. C. D. E. oropharynx oral vestibule pharyngeal recess laryngopharynx oral cavity propber

63.

The 28 YO female has come to you complaining hoarsely of difficulty swallowing (especially liquids) and in speaking. As you performed the physical examination, you observe that touching the posterior 1/3 of her tongue or pharyngeal wall on the right side produced no noticeable result. When the contralateral areas were stimulated, however, there was a distinct gag reflex and the uvula moved distinctly toward the left side of the pharynx. Use of a laryngoscope in examination of the true vocal folds would confirm that the patient had paralysis of the ... A. B. C. right vagus and right glossopharyngeal nerves right glossopharyngeal and left vagus nerves left vagus nerve only

D. E. 64.

right glossopharyngeal nerve only left vagus and left glossopharyngeal nerves

A staphylococcal infection that spreads from a region of abscess of a mandibular tooth into the soft tissues of the floor of the mouth and neck can result in suffocation, if antibiotic therapy is not instituted promptly. This clinical problem is referred to as ... A. B. C. D. E. ranula torus mandibularis submaxillary cellulitis (Ludwig's angina) quinsy ankyloglossia

65.

Imaging studies demonstrated that the patient's prostatic cancer had spread upward through the vertebral canal by way of the venous plexus of Batson in the epidural space. A metastatic tumor had grown in the posterior cranial fossa, there affecting cranial nerves leaving the skull. The patient's left shoulder drooped lower than the right shoulder. When protruded, his tongue deviates to the left. He has weakness in turning his head to the right. From this evidence, one can easily deduce which nerves have been affected: A. B. C4 C. D. E. right cranial nerves IX, X, XI and XII left cranial nerves X, XII and right XI, together with cervical nerves C3 and right cranial nerves X, XI, XII left cranial nerves XI and XII right cranial nerve XI, left cranial nerve XII

66. The patient responds properly as you check the blink reflex (corneal reflex) in both eyes, closing and opening both eyes. But, as you examine the patient's right eye with a small, bright flashlight, the right pupil does not constrict in response to the light, although the contralateral pupil does. You would be CORRECT in thinking that ... A. B. C. D. E. the patient obviously suffers from glaucoma in the right eye. the left oculomotor nerve is not functioning properly. the parasympathetic fibers in the right oculomotor nerve are not functional. the patient is blind in the right eye. the right oculomotor nerve is paralyzed.

67.

He had sustained a sharp blow to his right eye from the soccer ball, kicked with great force by the man facing him, and fell to the ground in pain. At the hospital, the eye was gently checked for injury and the ability to move it. The eye was adducted; the pupil could be turned slightly upward or downward slightly from that position, but he could not abduct the eye. It could therefore be rightly assumed that ... A. the trochlea had been torn from its attachment to the upper medial orbital wall. B. the lower division of the oculomotor nerve had been injured. C. the force had fractured the lamina papyracea, with entrapment of the medial rectus. D. he had suffered a break in the orbital plate of the maxilla and the inferior rectus was caught in the crack of the bone. E. the abducens nerve had been severed (cut) at the superior orbital fissure.

68.

On examining a lateral X-ray of the neck, one can usually identify the faint image of the hyoid bone at the level of the ... A. B. C. D. E. fourth cervical vertebra first cervical vertebra second cervical vertebra third cervical vertebra. fifth cervical vertebra

69.

Tumors or infections that develop in the middle ear cavity can affect structures associated with the various walls of the chamber. An infectious process that invades the promontory of the medial wall would MOST LIKELY involve the ... A. B. C. D. E. internal carotid artery in the carotid canal pyramid of the stapedius muscle temporal lobe of the brain jugular bulb cochlea

70.

The patient had suffered a hemisection of the lower part of the cervical portion of the spinal cord (approximately C6, 7, 8). As a result, one would expect to see ... A. B. C. D. E. complete ptosis of the ipsilateral superior palpebra decrease in sweating on the ipsilateral side of the face paralysis of the ipsilateral hemidiaphragm loss of the contralateral abdominal reflex a Babinski sign in the contralateral foot

41.

A four year old girl, Cherylee, was brought to your office because of frequently occurring colds and sore throat. Her mother told you that Cherylee was sick far more often than her two older brothers and her younger sister. When you asked the mother if the child had ever had any other problems, she mentioned that when Cherylee was born she had a connection between her "heart artery" and her "lung artery," and you realized that she was referring to the presence of a patent ductus arteriosus in the newly born baby. With this information, plus other evidence that Cherylee seems to be immunologically compromised, you correctly deduce that the child ... A. has the characteristics of Treacher Collins syndrome. B. is suffering from malar hypoplasia. C. possesses the Robin sequence, with resultant severe defects of the first arch. D. has abnormal development of the 1st pharyngeal pouch. E. suffers abnormal development of 3rd and 4th pharyngeal pouch derivatives.

42.

A tumor that invades the jugular canal is MOST LIKELY to result in A. B. C. D. E. hyperacusis hoarseness deafness a paralyzed and atrophied tongue loss of voluntary closure of the eyelids

43.

In the performance of the physical examination of the patient after the automobile accident, in which the woman had hit her forehead against the dashboard of the vehicle, you observe that her right pupil cannot be turned medially and downward. [She can adduct the affected eye medially toward the nose, but she cannot direct the pupil downward from that position.] She has no difficulty looking upward or laterally. If she first gazes laterally with the affected eye, she can look downward with it. It is probable from these signs that she has a nerve lesion or paralysis of the ... A. B. C. D. E. right abducens nerve right inferior rectus muscle right inferior oblique muscle right oculomotor nerve right trochlear nerve

44.

The eighth grade boy was observed in the clinic because of a midline, fluid-filled swelling just below the center of the hyoid bone. After palpating the soft mass, and suspecting its origin, you ask the boy to stick out his tongue. When he does this, you observe that the cervical mass also moves. It also moves when he swallows. This confirms your diagnosis that the lad has... A. B. an internal branchial cyst a cervical branchial cyst

C. D. E. 45.

a pharyngeal fistula aberrant thymic tissue a thyroglossal duct cyst

In checking the integrity of the cranial nerves, the examiner will typically check the major actions of the muscles of mastication. Which of the following muscles is of major importance both in opening the mouth and for protruding the jaw? A. B. C. D. E. Temporalis Masseter Lateral pterygoid Medial pterygoid Buccinator

46.

The cranial nerve that would be affected FIRST in cavernous sinus thrombosis, due to the passage of the nerve through the sinus, separated from its contents only by a thin layer of endothelium, is the ... A. B. C. D. E. Ophthalmic nerve Maxillary nerve Mandibular nerve Abducens nerve Optic nerve

47.

The 2-year-old child suffered major congenital problems of the first branchial (pharyngeal) arch. Structures that would MOST READILY be affected in such a case would be the ... A. B. C. D. E. malleus and incus stapedius muscle stylopharyngeus muscle vagus nerve stylohyoid ligament

48.

The 27-year-old woman underwent two essentially identical operations in which her stellate ganglia and adjacent segments of her sympathetic chains were excised. As a result, the chronic problem of vasoconstriction in her upper limbs had improved greatly. As a consequence of this procedure, however, she must expect ... A. excessive sweating from her facial skin, especially in response to appetizing olfactory stimuli B. smoothing of the nasolabial folds C. decreased ability to close her eyelids tightly D. reduced ability to dilate her pupils, as needed in conditions of darkness E. decreased secretions of the mucous glands in her trachea and primary bronchi

49.

The 10 month old daughter was brought by her mother to the pediatrician after she discovered that when the child began to stand up, one leg appeared shorter than the other. The physician diagnosed the child's condition as congenital dislocation of the hip. This condition ... A. B. C. D. E. usually results from malformations of the acetabulum (hip socket), with dislocation occurring after birth is attributable to excessive tightness of the articular capsule of the hip joint, which pulls the limb into a chronic state of abduction always occurs in utero is also called amelia does not require any treatment for the child to walk normally

50.

The 25-year-old, 3rd semester medical student was hit in the left eye by the football, which had just been thrown with great force by the former professional quarterback on the opposite team. The football lacerated the upper eyelid, which was closed at the time of impact, compressing the eyeball with force. On examination, you discover a severe hematoma (bruise) of the upper lid, and echymoses (broken blood vessels) in the lateral portion of the sclera of the eye. The iris of the left eye appears sunken in comparison to the right. When you test extraocular eye function, the patient cannot look up with the left eye when it is abducted, or look up when the left eye is adducted. What is the MOST LIKELY diagnosis? A. Direct damage to the inferior division of CN III and CN IV B. Blow-cut fracture and entrapment of inferior rectus and inferior oblique muscles C. Direct damage to the superior division of CN III and CN VI D. Fracture to the greater wing of the sphenoid bone and lateral orbital wall and entrapment of lateral rectus and inferior oblique muscles E. Direct damage to the zygomaticofacial branch of CN VII and ophthalmic division of CN V

51.

Shortly after an injured lineman was carried from the football field, the quarterback was tackled by a very large second semester medical student playing the position of linebacker. The quarterback landed heavily on his left shoulder, resulting in great pain and his departure from the game. Upon examination, it was noted that the patient could not abduct his left arm, nor could he rotate the arm laterally. He could not flex his elbow strongly, either, although he could extend it, after it had been flexed by the student health physician. He had no difficulty flexing or extending his fingers, nor performing fine movements with his fingers. The "accident" obviously injured, or traumatized, the A. B. C. D. E. radial nerve upper trunk of the brachial plexus lower trunk of the brachial plexus posterior cord of the brachial plexus, independent of other structures the medial and lateral roots of the median nerve

52.

The patient required excision of a mass in the floor of his mouth. Unfortunately the extent of the tumor resulted in postoperative problems with function of the tongue. When he was asked to protrude ("stick out") his tongue, it pointed to the patient's right; he could not point it straight ahead, nor to the left. The structure that has been paralyzed is the ... A. B. C. D. E. right vagus nerve left hyoglossus muscle the left hypoglossal nerve the right genioglossus muscle the left lingual nerve

53.

The prominent US political leader remembered that he had hit his head on a low- hanging branch of a tree while riding horseback, although he had not fallen from the saddle. The confusion and memory problems he was experiencing recently resulted in cranial examination with magnetic resonance and CT imaging. A subdural hematoma was diagnosed. Such an injury resulted from injury to ... A. B. C. D. E. the anterior communicating artery middle meningeal artery inferior petrosal venous sinus a parietal emissary vein, passing into the diploe from the scalp a cerebral vein

54.

The 35-year-old man presented with aching pain in his right upper teeth and a sensation of fullness and pulsation in his right cheek. Examinations reveal pus in his right middle nasal meatus. These symptoms are MOST suggestive of ... A. B. C. D. E. superior ethmoidal sinusitis maxillary sinusitis inflammation of the nasolacrimal duct sphenoidal sinusitis frontonasal sinusitis

55.

Which of the following events might be expected with laceration (cutting or tearing) of the middle meningeal artery at the region of the pterion? A. Interruption of arterial blood supply to the frontal lobe of the brain on the affected side B. Epidural hematoma, increased intracranial pressure C. Detection of blood in the cerebrospinal fluid D. Subdural hematoma, associated with herniation of the brainstem E. Subarachnoid hemorrhage and spasm of cerebral arteries, followed by ischemia

56.

One of the hazards in thyroid surgery is the possibility of injury to the recurrent laryngeal nerves, with resulting paralysis of the muscles that open the airway, separating the vocal cords. The muscles which perform this function are the ... A. B. C. D. E. posterior cricoarytenoids cricothyroids transverse and oblique fibers of the arytenoideus lateral cricoarytenoids thyroarytenoids

57.

The patient, the manager of a small convenience store, had suffered a gunshot wound to the right shoulder. The axillary artery was torn between the second and third parts of the artery. To avoid excessive blood loss, vascular clamps were applied to the damaged vessel until a vascular surgeon could arrive at the small community hospital to repair the site of injury. While waiting for the surgical repair, arterial blood flow to the limb would depend to a considerable extent upon anastomoses between ... A. B. C. D. E. intercostal branches of the internal thoracic and the lateral thoracic artery the transverse cervical artery and the suprascapular artery the profunda brachii artery and the subscapular artery the suprascapular artery and the circumflex scapular artery thoracodorsal artery and the circumflex scapular artery

58.

A child is presented by his mother in the morning clinic. The young boy has a unilateral cleft lip, and the mother asks you for your advice on the proper course of corrective treatment. With your excellent grasp of embryology, you first remember that this defect results usually from a... A. B. C. D. E. failure of fusion of the mandibular prominences failure of fusion of the medial nasal swelling with the maxillary process persistence of the nasolacrimal groove failure of fusion of the lateral nasal swellings failure of fusion of the maxillary shelves (swellings) with each other

59.

During the course of a total physical examination in the Geriatrics division of the hospital, including Magnetic Resonance Imaging, the images provided evidence - of the presence of a small (2.0 cm) tumor which involved several nerves in the cranial cavity of the 67-year-old male patient. His right shoulder drooped, in comparison with the left shoulder. He could not abduct the right arm above 90 degrees. He had difficulty swallowing. He had weakness turning his head to the left. On oral exam, his uvula pointed to the left, and sensitivity of his pharynx to stimuli was reduced. Lesions of which combination of nerves would BEST explain the signs? A. B. C. D. Right vagus, right spinal accessory, right glossopharyngeal Left vagus, left spinal accessory, right glossopharyngeal Right mandibular (V3), left glossopharyngeal, left superior laryngeal Right recurrent laryngeal, right spinal accessory, right hypoglossal

E. Left recurrent laryngeal, left spinal accessory, left pharyngeal branch of vagus 60. Intralaryngeal structures are involved frequently in traumatic injuries, stress from overuse, infectious processes and other pathology. Which of the following statements is TRUE regarding laryngeal anatomy? A. B. C. D. E. 61. The space beneath the true vocal cord or ligament is drained by specific lymphatic vessels which drain directly to the prelaryngeal node (Delphian node) or to the juguloomohyoid nodes, bilaterally. The rima glottidis is the space between the true vocal folds and the false vocal folds. Sensory supply to the interior of the larynx from true vocal folds to vestibule is provided by the recurrent laryngeal nerves. The glottis consists of the rima glottidis and the true vocal folds. The vocal process of the corniculate cartilage provides attachment for the true vocal ligament.

The young patient was to undergo a tonsillectomy for removal of the palatine tonsils. The pediatric surgeon, performing this procedure by himself for the first time, reviewed the anatomy of this area. He should remember .... A. B. C. D. E. the palatine tonsil is found within the oropharynx. the glossopharyngeal nerve passes deep to the lymphoid tissue of the tonsillar fossa. the principal source of arterial supply to the palatine tonsil arises from the facial artery. the lymphoid tissue to be removed lies just behind the structure that forms the anterior pillar of the fauces. all of the statements listed in the choices of this question are true.

62.

At your rural clinic a five-year-old girl is brought in by her parents because she has a "wry neck." Physical examination reveals no increased tone in neck muscles and all cranial nerve tests are normal. Her history reveals a difficult breech ("feetfirst") birth, however, during which excessive traction was put on her head and neck. You diagnose Congenital Torticollis. Damage to what structure(s) accounts for this condition? A. B. C. D. E. Ansa cervicalis Sternocleidomastoid muscle Posterior scalene muscle Ligamentum nuchae Anterior scalene muscles

63.

A 71-year-old woman is brought to the hospital complaining of difficulty in breathing. The attending physician decides that she should be given an emergency tracheostomy, but when the midline incision is made through the skin below the thyroid cartilage, profuse bleeding occurs. After tying off the bleeding vessel and inserting the tracheal tube, the physician then remembers

(too late) that in some 10% of individuals a variable artery lying in the midline below the thyroid gland can be damaged during tracheostomy. The vessel can arise from the aortic arch, the vertebral artery, or the common carotid artery. This vessel is the A. B. C. D. E. 64. ascending cervical artery middle thyroid artery ascending pharyngeal artery thyroid ima artery superior laryngeal artery

A 14-year-old male with severe acne is brought into your office with a painful inflammation of the right side of his face, below the eye. He is feverish and complains of headache. Laboratory tests reveal elevated white cell counts in serum and cerebrospinal fluid, indicative of spread of bacterial infection into the intracranial space. The anatomical route through which the infection spread is MOST LIKELY the ... A. facial vein to internal jugular vein to inferior petrosal sinus B. transverse cervical vein to pharyngeal venous plexus to basilar venous plexus C. angular vein directly to superior petrosal vein to transverse venous sinus D. angular vein to ophthalmic vein to cavernous sinus (or, anterior intercavernous sinus) E. transverse facial vein to retromandibular vein and directly to middle meningeal vein

65.

Your 15-year-old male patient arrives at your office complaining of a "sprained shoulder" which he reports occurred while learning to serve in tennis. His symptoms include diffuse tenderness in the root of the neck on the right side, pain in the right shoulder exacerbated by movement of his arm and deep breathing, and tingling and numbness in his right arm. An x-ray of the region reveals the presence of a right cervical rib, and you diagnose Thoracic Outlet Syndrome. What anatomical structures have been compressed? A. B. C. D. E. Subclavian vein, cupola of lung, and spinal accessory nerve Anterior scalene muscle, phrenic nerve and cervical plexus Subclavian artery and brachial plexus Deep cervical artery, supraclavicular nerves and ansa cervicalis nerve Supreme (highest) intercostal artery, long thoracic nerve and intercostobrachial nerve

66.

The fascia of the neck are important in spread of infections, blood, anesthetic agents and also in operative approaches. Among the characteristics of the superficial investing fascia is that it ... A. encloses the platysma muscle B. covers, and surrounds the viscera of the neck, including the larynx and pharynx

C. covers the ventral surface of the anterior scalene muscle and long thoracic nerve D. provides the anterior boundary of the "danger space" of the neck E. ensheathes (surrounds) and passes between the trapezius and the sternocleidomastoid muscles 67. A 7-year-old girl is brought into your pediatric clinic coughing, and with extreme pain in her throat. She had been playing dressup with her dolls and swallowed an open, small safety pin, which had lodged somewhere in her throat. With a laryngoscope, the pin is seen in the laryngopharynx, inferior and lateral to the right aryepiglottic fold. The probable location of the pin, and the nerve conveying the sense of pain, are ... A. piriform recess; pain conveyed by the internal laryngeal nerve B. right vallecula; pain fibers of the glossopharyngeal nerve C. subglottic space; pain carried by the recurrent laryngeal nerve D. palatopharyngeal fold and muscle; sensory fibers of the pharyngeal branch of V3 E. laryngeal ventricle; sensation carried by the glossopharyngeal nerve 68. The trochlear and oculomotor nerves can be affected directly by aneurysms of either of two arteries as the nerves leave the brainstem, for both nerves pass between the two arteries. One of these two arteries contributes directly to the "arterial circle of Willis"; the other does not. The artery that contributes to the "Circle" and which can affect either of the two nerves is the ... A. B. C. D. E. 69. anterior cerebral posterior cerebral middle cerebral superior cerebellar anterior inferior cerebellar

Several attempts had been made to stop the bleeding from Kiesselbach's area of the patient's nose, including application of silver nitrate and cauterization. At times, she was in danger of fatal aspiration of blood. The bleeding was stopped finally by a combination of anterior and posterior nosepacks. Epistaxis (nosebleed) can be an important source of loss of blood, when it cannot be stopped. This is due in large part to the number of arteries that supply the area of the nasal septum. Arteries which provide branches to the anterior inferior part of the nasal septum include the ... A. branches of the sphenopalatine artery and a branch derived from the facial artery B. posterior ethmoidal artery, branches of the lesser palatine C. ascending pharyngeal artery, dorsal branches of the lingual artery D. infraorbital artery branches, lateral nasal branches of angular artery E. supratrochlear arterial branches, pharyngeal branches of maxillary artery and posterior ethmoidal

70.

Congenital clubfoot was diagnosed in the young child. In this condition, which of the following statements is CORRECT? A. B. C. D. E. All of the bones of the foot have fused together in the shape of a club Congenital clubfoot involves the talus bone of the ankle The foot is always everted (sole turned outward) in clubfoot Clubfoot is also called meromelia In congenital clubfoot, the calcaneus (the heelbone) is absent

71.

The patient was brought to the emergency room coughing violently, and complaining that she had a bug in her ear. On examination, a large, common household insect was found lodged in her ear. After forceps removal of the creature and bathing of the external ear canal, together with the administration of a sedative, the patient's violent coughing ceased. The nerve fibers carrying the sensory input from the ear canal irritation, AND the nucleus or ganglion containing the motor neurons responsible for the spasmodic contraction of pharyngeal and laryngeal musculature were the ... A. auricular branches of the vagus and the nucleus ambiguus B. branches of the auriculotemporal nerve and the trigeminal ganglion C. sensory fibers of the glossopharyngeal nerve and the inferior salivatory nucleus D. posterior auricular branches of the facial nerve and the geniculate ganglion E. pharyngeal branches of the vagus nerve and the nucleus solitarius

72.

The 4th semester student was performing a physical examination of a 19-year-old female patient from Portsmouth who was troubled by a problem with cranial nerve dysfunction. The student first asked the patient to turn her head to the left, and look to the left. She-then gently touched the limbus of the patient's right eye with a little bit of cotton. As she touched the eye, she observed that the patient's right eye blinked, but the left one didn't. She then repeated the procedure, having the young woman turn her head and her eyes to the right, while the student stroked the left limbus. Now, she noted that the right eye blinked, but the left one did not. She correctly deduced from her observations that ... A. B. C. D. E. the patient's eyes showed normal responses to sensory stimuli the patient exhibited paralysis of the left facial nerve it was clear that the patient had bilateral weakness of the levator palpebrae the young woman had paralysis of her left ophthalmic nerve the patient had obvious conduction problems with her right ophthalmic nerve

73.

A young man is brought to the emergency department with an icepick protruding from the lower part of his neck, just above the junction of his left clavicle with the manubrium. The icepick had passed directly posteriorly from the point of entry, emerging through his left trapezius muscle just above the superior angle of the scapula. Before attempting to extract the slender weapon, the intern reviewed the structures that could be injured at the site of the entry wound. One structure he realized immediately WOULD NOT be injured, because of the location of the wound: A. B. C. D. E. the apex of the lung the internal jugular vein the thoracic lymphatic duct the subclavian vein the left inferior thyroid vein

31.

A 45-year-old man visits his doctor because of ringing in his left ear. Physical examination reveals an infection of the middle ear. If the infection spreads into the facial canal located in the medial wall of the tympanic cavity, which of the following signs and symptoms is LEAST LIKELY to occur? A. B. C. D. E. Vertigo (dizziness) Loss of lacrimation Reduced salivation Loss of taste from tip of tongue Sounds are annoyingly loud

32. A young boy is taken to the pediatrician because of a nose bleed. Examination of Little's area shows blood exuding from the major artery that supplies the nasal cavity, which is the: a. b. c. d. e.
33.

Greater palatine Sphenopalatine Anterior ethmoidal Posterior ethmoidal Superior labial

A 35-year-old woman suffers from maxillary sinusitis. Pus generated in the maxillary sinus is MOST LIKELY to drain into what part of the nasal cavity? a. b. c. d. e. Inferior meatus Sphenoethmoidal recess Superior meatus Middle meatus Vestibule

34.

A teenager is diagnosed with cavernous sinus thrombosis. Infected blood drains from the cavernous sinus directly into the: a. b. c. d. e. Sigmoid sinus Straight sinus Occipital sinus Superior and inferior petrosal sinuses Internal jugular vein

35.

A 28-year-old biker received a hard blow to the side of his head. X-ray revealed pooling of blood in his epidural space. The source of blood is MOST LIKELY a ruptured: a. b. c. d. e. Middle meningeal artery Emissary vein Cerebral vein Cerebral aneurysm Cerebral artery

36. A 55-year-old woman presents to her doctor with slight ptosis (drooping) of her left upper eyelid and mydriasis (dilation of pupil). A diagnosis of ophthalmoplegia (inability to move the eye) is made. This ptosis is due to: a. b. c. d. e.
37.

Denervation of the smooth muscle component of levator palpebrae superioris Compression of the pterygopalatine ganglion Denervation of orbicularis oculi Injury to the facial nerve Injury to the oculomotor nerve

Increased Cerebral Spinal Fluid pressure that compresses the contents of the optic nerve is MOST LIKELY to cause: a. b. c. d. e. Swelling of the optic disc or papilla Impaired vision Glaucoma Total blindness Degeneration of the retina

38. A 14-year-old girl with a bad case of acne complains of severe headache, fever, tingling and burning in the skin of the midface and loss of all voluntary movement of her right eye. Physical and ophthalmoscopic examinations reveal swollen eye and engorged retinal veins. The loss of function of the right extraocular eye muscles is due to involvement of which structures that pass through the cavernous sinus? a. CNIV and CNVI b. CNVII and CNVI c. CNIII, CNIV and CNV1 d. CNV1, CNV2 and CNVIII e. CNIII, CNIV and CNV
39.

A 24-year-old football player is referred to an orthopedic surgeon because of a mid-shaft fracture of the humerus. Surgery reveals a hematoma that resulted from an artery that was torn by a bone fragment. This artery is MOST LIKELY the: a. b. c. d. e. Profunda brachii (deep brachial) Radial Axillary Brachial Radial collateral

40.

A 50-year-old woman recently found out that she has carpal tunnel syndrome that resulted from dislocation of a carpal bone. Which carpal bone is MOST LIKELY to dislocate? a. b. c. d. Hamate Capitate Scaphoid Pisiform

e. Lunate 41. After a tonsillectomy your 14-year-old female patient has persistent bleeding from the tonsillar bed. Blood flow is slow but constant. The source of the blood is: a. b. c. d. e. Tonsillar branch of the facial artery External palatine vein Ascending palatine artery Superior thyroid vein Ascending pharyngeal artery

42. A 62-year-old woman who was a passenger in a car involved in a multi-vehicle accident suffered cuts and bruises on her face and a brain concussion when her head forcibly hit the inside door frame of the car. On neurological examination she was found to have no sensation over the posterior one-third of her tongue and her gag response was absent. Cranial x-ray revealed she had a fracture of the cranial base. The fracture went through what structure? a. b. c. d. e. Foramen spinosum Cavernous sinus Carotid canal Jugular foramen Hypoglossal canal

43. A 22-year-old male comes into your office complaining of hoarseness and an unsettling inability to change the tone of his voice normally. He has never smoked and he drinks alcohol in moderation. He is of normal weight and has no nasal, oral, or pharyngeal obstructions. When you examine him with the laryngoscope you note that the laryngopharyngeal mucosa is normal, all movements of the arytenoid cartilages seem normal, and the vocal folds are free of inflammation and polyps. You do notice an asymmetrical dislocation of the rima glottidis to the left when the patient attempts to speak in a higher tone of voice, indicating that the forward rotation of the thyroid cartilage is weakened. The only noteworthy point of the patient's history is that he is a kick boxer and sustained a blow to the right side of his throat the night before he noted the alterations in his voice. Damage to which nerve is responsible for his symptoms? a. b. c. d. e. Pharyngeal plexus Internal laryngeal nerve External laryngeal nerve Recurrent laryngeal nerve Glossopharyngeal nerve

44. An 11-year-old female was brought into the Emergency Room complaining of a fish bone stuck in her throat. She was accompanied by her father who explained that the family had been at a fish barbeque when his daughter had suddenly grabbed her throat and started coughing violently. Coughing had not dislodged the bone, but she was able to breathe without difficulty. Any attempt to swallow, however, was painful because of the obstruction. Where is the bone likely to be lodged?

a. b. c. d. e.
45.

The laryngeal ventricle The piriform recess The vallecula The laryngeal sinus The scaphoid fossa

A 34-year-old female patient who had been prescribed eye drops for a seasonal allergy complains to her physician that she tastes the drops every time she uses them. Through which of the following mechanisms did the patient MOST LIKELY taste the eye drops? a. Drainage into the oropharynx via a pharyngeal fistula and sensation by taste buds on the posterior third of the tongue b. Drainage through the nasolacrimal duct to pharyngeal taste buds c. Drainage through the frontonasal duct to epiglottic taste buds d. Drainage into the nasal cavity and sensation via the vomeronasal organ e. Drainage into the oral cavity via the incisive foramen and sensation by taste buds on the anterior two-thirds of the tongue

46. During the surgical removal of a cancerous thyroid gland in a 71-year-old female patient, her left recurrent laryngeal nerve is accidentally severed. The effect(s) will be: a. Need for artificial respiratory support because of the inability to abduct the vocal folds b. Hoarseness c. Loss of the cough reflex because of denervation of the larynx above the vocal folds d. Inability to speak because of paralysis of all the intrinsic laryngeal muscles (except cricothyroideus) e. Inability to swallow due to paralysis of the pharyngeal constrictors 47. During childbirth, an expectant mother is admonished to push. What combination of muscles does the mother use to perform this Valsalva maneuver and increase her intra-abdominal pressure? a. b. c. d. Pectoralis minor and major, stylopharyngeus, and scalenes Thyroarytenoid, cricothyroid, and trapezius Sternocleidomastoid, scalenes, and cricothyroid Serratus posterior superior and inferior, external intercostals, and posterior cricoarytenoid e. Lateral cricoarytenoid, arytenoids, and external oblique 48. An emergency cricothyroidostomy is performed a. b. c. d. e. Through the conus elasticus of the larynx Superior to the vocal folds but inferior to the vestibular folds In the midline through the median cricothyroid ligament Inferior to the cricoid cartilage Through the lamina of the thyroid cartilage

49. Which of the following structures of the adult head and neck are derivatives of the third pharyngeal arch? a. Superior horn of the thyroid cartilage, aryepiglottic folds, and cuneiform cartilages b. Epiglottis, palatapharyngeus muscle, and superior pharyngeal constrictor muscles c. Pterygomandibular raphe, tensor veli palatini muscle, and anterior belly of the digastric d. Lesser horn of the hyoid bone, stapedius muscles, and stylohyoid ligament e. Greater horn of the hyoid bone, stylopharyngeus muscles, and lateral glossoepiglottic folds 50. One muscle is responsible both for protraction and retraction of the tongue. This muscle is: a. b. c. d. e. Mylohyoid Genioglossus Palatoglossus Hyoglossus Styloglossus

51. During swallowing which muscle exerts pull on the wall of the auditory tube, effecting equalization of pressure between nasopharynx and middle ear cavity? a. b. c. d. e.
52.

Superior pharyngeal constrictor Salpingopharyngeus Tensor tympani Stapedius Musculus uvulae

A third year medical student was asked to perform venipuncture on a patient's right arm. He is initially unsuccessful, and he changes the site of puncture to a slightly more medial one. The patient experiences pain radiating down the ventral surface of the forearm and hand, including the thumb, index finger and middle finger. The nerve penetrated is MOST LIKELY: a. b. c. d. e. The medial antebrachial cutaneous nerve The median nerve The radial nerve A branch of the musculocutaneous nerve The ulnar nerve

53. An occlusion of the axillary artery does not usually result in total loss of blood supply to the upper limb because of anastomoses around the scapula. Arteries which arise from the axillary artery and contribute in a significant way to these anastomoses include the: a. b. c. d. e. Suprascapular artery Deep brachial artery (profunda brachii) Subscapular artery Transverse cervical artery Dorsal scapular artery

54. A fracture of the mid-portion of the shaft of the humerus had resulted in injury to the nerve lying in the spiral groove of the bone. With injury to this nerve, what effects would be expected in the functions of the affected limb? a. b. c. d. e.
55.

"Wristdrop" would occur The triceps brachii would still be functional There would be absence of the brachioradialis reflex There would be weakness in the grip of the flexor muscles of the fingers All of the responses listed would be expected

Which of the following nerves DOES NOT NORMALLY have any contribution from spinal nerve C7? a. b. c. d. e. Thoracodorsal nerve Medial pectoral nerve Musculocutaneous nerve Long thoracic nerve Radial nerve

56.

A patient presents with an Erb-Duchenne palsy. Which of the following would NOT be affected? A. B. C. D. E. Infraspinatus muscle Biceps brachii muscle Subclavius muscle Teres minor muscle Pectoralis minor muscle

57.

A patient presents to the office with a loss of sensation over the lateral portion of the palm and the lateral three digits. The patient has difficulty with fine motor movements of the hand (i.e. buttoning her shirt or making the "OK" sign). Which of the following nerves is affected? A. B. C. D. E. Musculocutaneous nerve Ulnar nerve Median nerve Radial nerve Thoracodorsal nerve

58.

A 17-year-old girl was thrown from her horse while attempting a difficult jump. She landed on her right shoulder and the right side of her head. It was noticed after a week of hospitalization that she kept her right arm medially rotated and close to her side with the forearm pronated. At physical examination, an area of anesthesia was found along the lateral side if the upper part of the arm. Which nerve(s) was(were) damaged during the accident? A. B. C. D. E. The axillary nerve The radial nerve The middle trunk of the brachial plexus The lower trunk of the brachial plexus The C5 and C6 roots of the brachial plexus

59.

The position adopted by the right arm in this patient can be explained by paralysis of which of the following groups of muscles? A. Flexor digitorium superficialis, flexor pollicis longus, flexor carpi radialis longus, and flexor carpi ulnaris B. Extensor carpi ulnaris, supinator, extensor indicis, and extensor digiti minimi C. Supraspinatus, deltoid, biceps brachii, greater part of brachialis, infraspinatus, and teres minor D. Latissimus dorsi, triceps, anconeus, brachioradialis and supinator E. Pectoralis major, abductor pollicis longus, extensor pollicis brevis, and extensor pollicis longus

60.

A 30-year-old plasterer was finishing a difficult ceiling in a remodeled kitchen. He was standing on top of a stepladder with his right arm above his head. As he moved the trowel filled with plaster across the ceiling in his right hand, he suddenly felt an acute spasm of pain over the tip of the right shoulder. At physical examination of the patient in the emergency department, it was found that the pain in the right shoulder recurred in the middle range of abduction and that there was tenderness over the greater tubercle of the humerus. What is the MOST LIKELY structural damage in this patient? A. B. C. D. E. Lesion of the rotator cuff Fracture of the upper end of the humerus Tearing of the deltoid muscle Fracture of the acromion Tearing of the trapezius muscle

41.

A 65-year-old man is involved in a road traffic accident. A CT scan shows he sustained an isolated fracture of the posterior cranial fossa involving the jugular foramen. Which of the following signs and symptoms is he MOST LIKELY to be suffering from? A. B. C. D. E. Deviation of his tongue on protrusion Loss of smell Exopthalmos Loss of hearing A sagging shoulder

42.

A prominent US political leader remembered that he had hit his head on a low-hanging branch of a tree while riding horseback, although he had not fallen from the saddle. The confusion and memory problems he was experiencing recently resulted in cranial examination with magnetic resonance and CT imaging. A subdural hematoma was diagnosed. Such an injury resulted from damage to A. B. C. D. E. middle meningeal artery inferior petrosal venous sinus a parietal emissary vein, passing into the diploe from the scalp the anterior communicating artery a cerebral vein

43.

Which of the following statements concerning the cubital fossa is CORRECT? A. B. C. D. E. The cephalic vein overlies the bicipital aponeurosis The floor is formed by the head of flexor digitorum superficialis The median nerve lies lateral to the brachial artery Contains the ulnar nerve The brachial artery lies immediately medial to tendon of the biceps

44.

In angina pectoris, the pain radiating down the left arm is associated with increased activity of sensory (afferent) fibers that travel along sympathetic pathways to reach the: A. B. C. D. E. Brain stem Upper four or five cervical spinal cord segments Upper four or five thoracic spinal cord segments Upper two lumbar spinal cord segments None of the choices are correct

45.

Which of the following group of nerves is intimately related to a portion of the humerus and is MOST LIKELY to be affected by fractures of the humerus? A. B. C. D. E. Axillary, median, ulnar Axillary, radial, ulnar Axillary, median, musculocutaneous Axillary, musculocutaneous, radial Median, radial, ulnar

46.

Which of the following is NOT TRUE of an indirect inguinal hernia? A. B. C. D. E. Passes lateral to the inferior epigastric artery Passes medial to the inferior epigastric artery Courses through the inguinal canal Enters the deep inguinal ring Is associated with an incompletely obliterated processus vaginalis

47.

The pampiniform plexus of veins on the right side empties into the: A. B. C. D. E. Left renal vein Inferior vena cava Right renal vein Inferior mesenteric vein Internal iliac vein.

48.

Lymph from the testes drains to which of the following lymph nodes? A. B. C. D. E. Deep inguinal lymph nodes Para-aortic (lumbar) lymph nodes External iliac lymph nodes Superficial inguinal lymph nodes Internal iliac lymph nodes

49.

The clinical signs associated with Horner's Syndrome such as loss of sweating (anhydrosis) and miosis (constriction of pupil) can BEST be explained in terms of a A. B. C. D. E. sympathetic nerve lesion parasympathetic nerve lesion branchial motor nerve lesion somatic motor nerve lesion reduced blood supply to head and neck structures

50.

A lesion to which one of the following structures, at the location given, would stop secretion from the lacrimal gland? A. B. C. D. E. Oculomotor nerve in the superior orbital fissure Glossopharyngeal nerve at the jugular foramen Otic parasympathetic ganglion in the infratemporal fossa Greater petrosal nerve in the petrous temporal bone Ciliary parasympathetic ganglion in the orbit

51.

Where are you MOST LIKELY to feel referred pain generated from an infected maxillary sinus? A. B. C. D. Upper teeth and skin of mid-face Forehead Anterior scalp Between eye and nose

E. 52.

Eyeball

Where are the cell bodies of the nerve that carries pain sensation from the TMJ? A. B. C. D. E. Nodose ganglion Jugular ganglion Trigeminal ganglion Geniculate ganglion Stellate ganglion

53.

A gunshot wound to the infratemporal fossa that affected general sensation to the anterior two-thirds of the tongue has MOST LIKELY injured the A. B. C. D. E. buccal nerve chorda tympani nerve lingual nerve inferior alveolar nerve muscular branches of V3

54.

Which of the following statements concerning sternocleidomastoid muscle is CORRECT? A. B. C. D. E. When contracted turns the face to the opposite side When contracted turns the face to the same side Is a strap muscle Is innervated by the cranial accessory nerve Attaches to the styloid process

55.

The internal carotid artery A. B. C. D. E. supplies the face and tongue enters the cranial cavity through the foramen lacerum has eight branches traverses the foramina transversaria of cervical vertebrae 1-6 passes through the cavernous sinus

56.

Regarding the head and neck, choose the CORRECT statement: A. B. C. D. E. Strap muscles are supplied by the supraclavicular nerves The hypoglossal nerve supplies taste to the tongue Temporalis elevates and retrudes the mandible The glossopharyngeal nerve is motor to the styloglossus muscle The nasolacrimal duct opens into the middle meatus of the nasal cavity

57.

A purulent infection can pass from your nasopharynx into your tympanic (middle ear) cavity by passing through the A. B. choana auditory (pharyngotympanic) tube

C. D. E. 58.

pharyngeal isthmus fauces posterior nares

Fractures of the cranial base can cause facial nerve palsy. With damage to the facial nerve, which of the following signs and symptoms are NOT LIKELY to be seen? A. B. C. D. E. Absence of saliva from the parotid gland Hyperacusis Weakness of the facial musculature Inability to produce tears Lack of secretion from the submandibular and sublingual salivary glands

59.

An infection that begins in scalp veins may spread intracranially into the dural venous sinuses by passing through A. B. C. D. E. Emissary veins Cerebral veins Meningeal veins Cerebellar veins Vertex veins

60.

Preganglionic parasympathetic fibers of the facial nerve, which have their cell bodies in the superior salivatory nucleus of the brainstem, are found in which branches of the facial nerve? A. B. C. D. E. Nerve to stapedius and greater petrosal nerve Temporal and zygomatic branches Zygomatic and buccal branches Greater petrosal and chorda tympani nerves Mandibular and cervical branches

61.

Choose the CORRECT order regarding the course of the facial nerve: I. II. III. IV. V. VI. A. B. C. D. E. Stylomastoid foramen Internal acoustic meatus Petrou$ temporal bone (facial canal) Brain-stem Posterior cranial fossa Parotid gland and face

IV, V, II, III, I and VI IV, V, I, III, II and VI IV, III, II, I and VI IV, III, If V, II and VI None of the choices are correct

62.

The primary function of the "rotator cuff muscles", is

A. B. C. D. E. 63.

flexion of the arm extension of the arm abduction of the arm rotation of arm stabilization of humerus in the glenoid cavity during movement of arm

In the "anatomical snuff box", the pulse felt would be that of: A. B. C. D. E. Dorsal carpal network Radial artery Ulnar artery Princeps pollicis artery osterior interosseous artery

64.

The artery is NOT a branch of the axillary artery: A. B. C. D. E. Lateral thoracic Subscapular Superior thoracic Suprascapular Thoracoacromial trunk

65.

A butcher who has a deep cut on his palm is unable to oppose his thumb. Which of the following nerves has MOST LIKELY been severed? A. B. C. D. E. Deep u1nar Superficial radial Recurrent motor branch of the median Superficial ulnar Deep radial

66.

A 70-year-old man visited his doctor after noticing his own voice sounded disturbingly loud (hyperacusis). Further examination revealed other symptoms such as inability to produce tears, impaired salivation, loss of taste from the anterior two-thirds of his tongue and paralysis of the buccinator, platysma and orbicularis oris muscles. A diagnosis of the facial nerve lesion was made. At what level has the facial nerve lesion MOST LIKELY occurred? A. B. C. D. E. Within the facial canal, just distal to the geniculate ganglion Within the facial canal, proximal to the origin of the chorda tympani nerve Within the facial canal, distal to the origin of the nerve to stapedius At the level of the stylomastoidforamen At the level of the internal acoustic meatus

67.

A young motorist visited her doctor three weeks after receiving severe lacerations to her parotid gland in a car accident. She had noticed that when she ate, the area of skin overlying the parotid gland sweated profusely. The doctor informed her that she had a condition known as Frey's syndrome. This syndrome was due to misdirection of the regenerating secretomotor parasympathetic fibers that supply the parotid gland that are present in the: A. B. C. D. E. Auriculotemporal nerve Chorda tympani nerve Facial nerve Greater superficial petrosal nerve Great auricular nerve

68.

A tumor that invades the jugular canal is MOST LIKELY to result in A. B. C. D. E. a paralyzed and atrophied tongue hyperacusis hoarseness deafness loss of voluntary closure of the eyelids

69.

In the performance of the physical examination of the patient after the automobile accident, in which the woman had hit her forehead against the dashboard of the vehicle, you observe that her right pupil cannot be turned medially and downward. (She can adduct the affected eye medially toward the nose, but she cannot direct the pupil downward from that position). She has no difficulty looking upward or laterally. If she first gazes laterally with the affected eye, she can look downward with it. It is probable from these signs that she has a nerve lesion or paralysis of the A. B. C. D. E. right inferior rectus muscle right inferior oblique muscle right oculomotor nerve right abducens nerve right trochlear nerve

70.

The eight-grade boy was observed in the clinic because of a midline, fluid-filled swelling just below the center of the hyoid bone. After palpating the soft mass, and suspecting its origin, you ask the boy to stick out his tongue. When he does this, you observe that the cervical mass also moves. It also moves when he swallows. This confirms your diagnosis that the lad has A. B. C. D. E. a pharyngeal fistula aberrant thymic tissue an internal branchial cyst a cervical branchial cyst a thyroglossal duct cyst

71.

In checking the integrity of the cranial nerves, the examiner will typically check the major actions of the muscles of mastication. Which of the following muscles is of major importance both in opening the mouth and for protruding the jaw? A. B. C. D. E. Medial pterygoid Temporalis Masseter Lateral pterygoid Buccinator

72.

The cranial nerve that MOST LIKELY would be affected FIRST in cavernous sinus thrombosis, due to the passage of the nerve through the sinus, separated from its contents only by a thin layer of endothelium, is the A. B. C. D. E. Maxillary nerve Mandibular nerve Abducens nerve Ophthalmic nerve Optic nerve

73.

A 27-year-old woman underwent two essentially identical operations in which her stellate ganglia and adjacent segments of her sympathetic chains were excised. As a result, the chronic problem of vasoconstriction in her upper limb had improved greatly. As a consequence of this procedure, however, she must expect A. B. C. D. E. decreased ability to close her eyelids tightly reduced ability to dilate her pupils, as needed in conditions of darkness excessive sweating from her facial skin, especially in response to appetizing olfactory stimuli smoothing of the nasolabial folds decreased secretions of the mucous glands in her trachea and primary bronchi

74.

A 35-year-old man presented with aching pain in his right upper teeth and a sensation of fullness and pulsation in his right cheek. Examinations reveal pus in his right middle nasal meatus. These symptoms are MOST suggestive of A. B. C. D. E. sphenoidal sinusitis superior ethmoidal sinusitis maxillary sinusitis inflammation of the nasolacrimal duct frontonasal sinusitis

75.

Which of the following events might be expected with laceration (cutting or tearing) of the middle meningeal artery at the region of the pterion? A. B. C. Epidural hematoma with increased intracranial pressure Detection of blood in the cerebrospinal fluid Subdural hematoma, associated with herniation of the brainstem

D. E. 76.

Interruption of arterial blood supply to the frontal lobe of the brain on the affected side Subarachnoid hemorrhage and spasm of cerebral arteries, followed by ischemia

One of the hazards in thyroid surgery is the possibility of injury to the recurrent laryngeal nerves, with resulting paralysis of the muscles that open the airway, separating the vocal cords. The muscles which perform this function are the A. B. C. D. E. transverse and oblique fibers of the arytenoideus lateral cricoarytenoids posterior cricoarytenoids cricothyroids thyroarytenoids

77.

During the course of a total physical examination in the Geriatrics division of the hospital, including magnetic Resonance Imaging, the images provided evidence of the presence of a small (2.0 cm)) tumor which involved several nerves in the cranial cavity of the 67-year-old male patient. His right shoulder drooped, in comparison with the left shoulder. He could not abduct the right arm above 90 degrees. He had difficulty swallowing. He had weakness turning his head to the left. On oral exam, his uvula pointed to the left, and sensitivity of his pharynx to stimuli was reduced. Lesions of which combination of nerves would BEST explain the signs? A. B. C. D. E. Right recurrent laryngeal, right spinal accessory, right hypoglossal Right vagus, right spinal accessory, right glossopharyngeal Left vagus, left spinal accessory, right glossopharyngeal Right mandibular V3, left glossopharyngeal, left superior laryngeal Left recurrent laryngeal, left spinal accessory, left pharyngeal branch of vagus

78.

A 71-year-old woman is brought to the hospital complaining of difficulty in breathing. The attending physician decides that she should be given an emergency tracheostomy, but when the midline incision is made through the skin below the thyroid cartilage, profuse bleeding occurs. After tying off the bleeding vessels and inserting the tracheal tube, the physician then remembers (too late) that in some 10% of individuals a variable artery lying in the midline below the thyroid gland can be damaged during tracheostomy. The vessel can arise from the brachiocephalic, right common carotid or subclavian artery. This vessel is the A. B. C. D. E. middle thyroid artery ascending pharyngeal artery thyroid ima artery ascending cervical artery superior laryngeal artery

79. A 14-year-old male with severe acne is brought into your office with a painful inflammation of the right side of his face, below the eye. He is feverish and complains of headache. Laboratory tests reveal elevated white cell counts in serum and cerebrospinal fluid, indicative of spread of bacterial infection into the intracranial space. The anatomical route through which the infection spread is MOST LIKELY the A. B. C. D. E. 80. angular vein directly to superior petrosal vein to transverse venous sinus angular vein to ophthalmic vein to cavernous sinus (or anterior intercavernous sinus) facial vein to internal jugular vein to inferior petrosal sinus transversal cervical vein to pharyngeal venous plexus to basilar venous plexus transverse facial vein to retromandibular vein and directly to middle meningeal vein

A 70-year-old man develops a tumor in the distal part of the scalenus anterior muscle with pressure on related structures. Which of the following are LEAST LIKELY? A. B. C. D. E. Diminished flow in axillary vessels Prominent suprascapular vessels Prominent transverse cervical vessels Prominent circumflex scapular vessels Prominent thoraco acrominal vessels

36.

Which of the following DOES NOT get motor supply from the Pharyngeal Plexus? A. B. C. D. E. Palatoglossus Musculus uvulae Levator veli palatini Tensor veli palatini Palatopharyngeus

37.

The path by which lymphatics normally drain the tongue include A. B. C. D. E. tip to supraclavicular; body to submental nodes tip to deep cervical nodes; body to submental nodes tip to jugulodigastric nodes; root to infraclavicular tip to submental nodes; body to deep cervical tip to apical axillary; body to parotid nodes

38.

Which of the following DOES NOT enter the orbit through the superior orbital fissure? A. B. C. D. E. Ophthalmic artery Abducens nerve Oculomotor nerve Frontal nerve Trochlear nerve

39.

The general sensory innervation of structures in the orbit is via the: A. B. C. D. E. Facial (VII) nerve Trigeminal (V) nerve Oculomotor (III) nerve Trochlear (IV) nerve Sympathetic system

40.

Damage to the facial nerve will result in hyperacusis due to paralysis of which muscle? A. B. C. D. E. Auricularis posterior Tensor tympani Stapedius Tensor palatini Salpingopharyngeus

41.

A 24-year-old man was diagnosed with Bell's (facial nerve) palsy. This is MOST LIKELY to cause: A. B. Inability to chew Atrophy and paralysis of tongue muscles

C. D. E. 42.

Inability to close the eyelid and drooling of saliva out of the affected side of the mouth Visual impairment in the eye on the affected side Inability to turn the neck

A 46-year-old woman consulted her physician about a firm painless lump in her left breast. During physical examination the physician felt a lump in the upper medial quadrant of the breast. He also observed dimpling and thickening of the skin in this quadrant. Palpation of the axilla did NOT reveal palpable lymph nodes. Malignancy would MOST LIKELY metastasize via: A. B. C. D. E. Inferior phrenic nodes Axillary nodes Anterior pectoral nodes Parasternal nodes Upper deep cervical nodes

43.

Which of the following statement is NOT TRUE concerning the anatomy of bronchopulmonary segments: A. B. Each is a subdivision of a lobe Each is supplied by a secondary bronchus and a branch of the pulmonary artery There are usually 10-bronchopulmonary segments present in the right

C. lung D. Each can be surgically removed without greatly interfering with the function of the remaining segments E. Each is separated from the adjacent segment by connective tissue and veins 44. What nerve DOES the superior laryngeal artery accompany into the larynx? A. B. C. D. E. 45. Recurrent laryngeal Vagus Descendens hypoglossal of ansa cervicalis The nerve to the cricothyroid muscle Internal laryngeal

A patient has a severely damaged radial nerve resulting from fracture of the lower third of the humerus. The patient will experience A. B. C. D. E. a sensory loss over the ventral aspect of the base of the thumb an inability to oppose the thumb a loss of wrist extension, leading to wrist drop a weakness in pronating the forearm an inability to abduct the fingers

46.

Which of the following group of nerves is intimately related to a portion of the humerus and can be affected by fracture of the humerus? A. B. C. D. E. Axillary, median, musculocutaneous Axillary, musculocutaneous, radial Axillary, median, ulnar Axillary, radial, ulnar Median, radial, ulnar

47.

A patient cannot abduct his arm. This inability results from a lesion of which of the following nerves? A. B. C. D. E. Thoracodorsal and upper subscapular Suprascapular and axillary Radial and lower subscapular Axillary and musculocutaneous Suprascapular and dorsal scapular

48.

Inability to supinate the forearm could result from an injury to which of the following pair of nerves? A. B. C. D. E. Axillary and radial Radial and musculocutaneous Suprascapular.and axillary Musculocutaneous and median Median and ulnar

49.

A patient complains of sensory loss over the anterior and the posterior surfaces of the medial third of the hand and the medial one and one-half fingers. Which of the following nerves is injured? A. B. C. D. E. ulnar Axillary Radial Median Musculocutaneous

50.

A patient enters your office with a posterior dislocation of the head of the humerus putting pressure on the structures within the quadrangular space. Which nerve is in danger of being damaged or impaired with this type of dislocation? A. B. C. a. E. Musculocutaneous nerve Median nerve Axillary nerve Radial nerve Ulnar nerve

51.

A young child presents with a fracture of the clavicle at the junction of the lateral and middle thirds. The medial and lateral fragments are tilted upward and downward, respectively. Medial displacement of the lateral fragment is caused by contraction of which of the following muscles? A. B. C. D. E. Sternocleidomastoid Trapezius Deltoid Pectoralis major Subclavius

52.

After falling from his bicycle, the 10-year-old boy sustained a scalp injury that was followed by the appearance of a "black eye" (swelling and purplish discoloration of the eyelids). Which layer of the scalp is the MOST LIKELY passageway for the flow of the blood into the eyelid? A. B. C, D. E. The loose connective tissue beneath the aponeurotic layer The space beneath the skin The dense connective tissue layer The space just deep to the pericranium The space between the superficial fascia and the aponeurotic layer

53.

A 16-year-old teenager presents to the doctor with high fever, severe headache, and a pus-filled pimple on the upper lip. An infection from the superficial face, (danger zone of the face), had MOST LIKELY spread from the facial vein into the cavernous sinus within the cranial cavity via the: A. B. C. D. E. Superior ophthalmic vein Diploic vein Retromandibular vein Maxillary vein Internal jugular vein

54.

A 42-year-old woman presents to her doctor complaining of a sagging right shoulder and inability to turn her head up and to the left. These symptoms are MOST LIKEY due to injury of the: A. B. C. D. E. Right spinal accessory nerve Supraclavicular nerves Vagus nerve Left spinal accessory nerve C3 and C4

55.

Which of the following statements is INCORRECT: A. B. C. D. Fracture of cribriform plate may give CSF leakage from nose Battles sign is indicative of basilar skull fracture On X-ray, skull sutures have straight dark edges Pterion-important landmark for middle meningeal artery

E. 56.

Otitis media can lead to mastoiditis

A patient was experiencing pain over the deltoid region extending down the lateral side of the arm. A myelogram, injection of an opaque medium into the cerebro8pinal fluid was performed. X-ray studies revealed that there was a compression of the fifth cervical spinal nerve. From this, it is CORRECT to make the following assumption: A. B. C. D. E. The disk or bony process that is compressing the nerve lies between the fourth and fifth cervical vertebrae If not corrected, there will be sensory loss to the thoracic wall at the level of the nipple The anterior longitudinal ligament is being pressed against the fifth thoracic nerve and, possibly, the adjacent portion of the spinal cord The contrast medium was injected into the subdural space The outline of the subarachnoid space would be obscured at the site of the compressed nerve

57.

A 26-year-old, muscular male had apparently traumatized his long thoracic nerve during hiking. The next day, as a result of the paralysis of this nerve, you noticed during examination: A. B. C. D. E. Weakness in protraction of scapula on affected side Paralysis of his trapezius muscle and weakness in elevating his shoulders Difficulty in breathing, because of paralysis of his diaphragm Inability to retract (or, pull back) his scapulae Inability to adduct his arms with his pectoralis major muscles

58.

A newborn had suffered a complete tear of the upper trunk of the brachial plexus by excessive traction on its head at birth. The result of such injury would eventually include: A. B. C. D. E. Paralysis of the abductors and adductors of the fingers Inability to abduct the arm at the shoulder joint Paralysis of opposition of the thumb to the other digits Loss of the triceps reflex Loss of sensation on the medial side of the arm

59.

An eighth grade boy was observed in the clinic because of a midline, fluid-filled swelling just below the center of the hyoid bone. After palpating the soft mass, and suspecting its origin, you ask the boy to stick out his tongue. When he does this, you observed that the cervical mass also moves. It also moves when he swallows. This confirms your diagnosis that the lad has: A. B. C. D. E. A cervical branchial cyst A pharyngeal fistula Aberzant thymic tissue An internal branchial cyst A thyroglossal duct cyst

60.

A 35-year-old man presented with aching pain in his right upper teeth and a sensation of fullness and pulsation in his right cheek. Examinations reveal pus in his right middle nasal meatus. These symptoms are MOST suggestive of: A. B. C. D. E. Inflammation of the nasolacrimal duct Sphenoidal sinusitis Superior ethmoidal sinusitis Maxillary sinusitis Frontonasal sinusitis

61.

A 24-year-old football player is referred to an orthopedic surgeon because of a mid-shaft fracture of the humerus. Surgery reveals a hematoma that resulted from an artery that was torn by a bone fragment. This artery is MOST LIKELY the: A. B. C. D. E. Profunda,brachii (deep brachial) Radial Axillary Brachial Radial collateral

62.

After a tonsillectomy a 14-year-old female patient has persistent bleeding from the tonsillar bed. Blood flow is slow but constant. The source of the blood is MOST LIKELY the: A. B. C. D. E. Superior thyroid vein Tonsillar branch of the facial artery External palatine vein Ascending palatine artery Ascending pharyngeal artery

63.

An occlusion of the axillary artery DOES NOT usually result in total loss of blood supply to the upper limb because of anastomoses around the scapula. Arteries which arise from the axillary artery and contribute in a significant way to these anastomoses include the: A. B. C. D. E. Suprascapular artery Deep brachial artery (profunda brachii) Transverse cervical artery Subscapular artery Dorsal scapular artery

64.

A fracture of the mid-portion of the shaft of the humerus resulted in injury to the nerve lying in the spiral groove of the bone. With injury to this nerve, what would you NOT expect to see at examination? A. B. "Wristdrop" would be apparent The triceps brachii would still be functional

C. D. E. 65.

There would be absence of the brachioradialis reflex There would be weakness in the grip of the flexor muscles of the fingers Total loss of supination of forearm and hand

A 40-year-old ex-convict was shot in the left forearm with the bullet entering through the dorsal aspect, a hand's breath below the elbow arresting on the radius. Patient was subsequently unable to extend the wrist and fingers but sensation remained normal in the entire limb. Which of the following would NOT have occurred? A. B. C. D. E. Damage to superficial branch of radial nerve Damage to deep branch of radial nerve Damage to extensor carpi radialis longus Damage to extensor digitorum communis Damage to extensor carpi radialis brevis

66.

A 70-year-old man developed a tumor in the distal part of the scalenus anterior muscle with pressure on related structures. Which of the following would NOT be found at examination of the patient? A. B. C. D. E. Prominent circumflex scapular vessels Diminished flow in axillary vessels Prominent suprascapular vessels Prominent transverse cervical vessels Prominent thoraco acromial vessels

67.

A young male fell and put his right hand and forearm through a window. A transverse cut occurred across the ventral aspect of the mid forearm, extending as deep as the osseous plane. On examination, there is-profuse arterial hemorrhage and he has difficulty flexing the 4th and 5th fingers as well as adducting the hand. Which of the following are NOT LIKELY findings? A. B. C. D. E. The laceration is on the lateral aspect of the ventral surface of right forearm The ulnar nerve is cut or damaged The flexor carpi ulnaris muscle is damaged The ulnar artery is severed The radial artery is intact

68.

Concerning the insertion of a central venous line, which of the following statements is INCORRECT: A. B. C. D. E. The needle must be inserted lateral to the carotid pulsation in the neck A CXR must be performed after the procedure to rule out pneumothorax The needle must be inserted medial to the carotid pulsation in the neck The subclavian or internal jugular veins may be used Vagus nerve damage is a possible complication

69.

Two weeks following a thyroidectomy, a 45-year-old opera singer complains that she has difficulty in changing the tone of her voice. You examine her larynx

with a laryngoscope and note that while the arytenoid cartilages move normally, the thyroid cartilage does not tilt forward when you ask her to change the tone of her voice. Clearly there has been damage to a nerve. With which artery DOES the damaged nerve run? A. B. C. D. E. 70. Internal laryngeal artery Inferior thyroid artery Superior laryngeal artery Inferior laryngeal artery Superior thyroid artery

The following abnormality (dotted white line) is noted on the X-ray of a woman who has a swelling in her neck and difficulty with breathing. The abnormality is MOST LIKELY to be:

A. B. C. D. E.

Retrosternal goitre Pancoast tumor Aortic aneurysm Esophageal carcinoma Branchial cyst

41.

A 24 YO football player is referred to an orthopedic surgeon because of a mid-shaft fracture of the humerus. Surgery reveals a hematoma on the posterior aspect of the arm that resulted from bleeding from an artery that was torn by a bone fragment. This artery is MOST LIKELY the A. B. C. D. E. radial axillary brachial profunda brachii (deep brachial) radial recurrent

42.

A 50 YO woman discovered that the cause of her carpal tunnel syndrome was the dislocation of a carpal bone. The MOST LIKELY bone involved would be the B. A. lunate pisiform. C. D. E. scaphoid hamate capitate

43.

A 60 YO butcher accidently slashed his wrist, injuring his ulnar nerve. Which of the following actions would be lost as a result of the injury? A. B. C. D. E. flexion of the distal phalanx of the fifth digit (little finger) extension of the thumb adduction of the fifth digit abduction of the thumb opposition of the thumb

44.

A 25 YO, male presents with a. mass in the anteromedial aspect, of his spermatic cord, and is told he has an indirect inguinal hernia. During surgery, a nerve is inadvertently cut which results in loss of sensation around the base of the penis and anterior scrotum. The severed nerve is MOST LIKELY the... A. B. C. D. E. iliohypogastric ilioinguinal genitofemoral posterior femoral cutaneous perineal

45.

To permit operating on a patient with severe diverticulosus of the transverse colon it would be necessary to first ligate (tie off) or clamp the source of arterial supply. In this case, the artery is MOST LIKELY to be the ... A. B. C. D. right colic artery left colic artery middle colic artery sigmoid arteries

E. 46.

superior mesenteric artery

The patient had been experiencing pain in the buttock and posterior thigh because part of his sciatic nerve passed through the muscle that forms the "key" of the gluteal region, and was being compressed. The muscle that was compressing the nerve was the... A. B. C. D. E. gluteus medius obturator internus piriformis obturator externus quadratus femoris

47. The proximal lateral aspect of the shoulder of the skier hit the outcropping of rock with considerable force. Upon examination with X-ray, no broken bones were seen, but the patient cannot lift the affected arm to the level of the shoulder and there is no sensation over the deltoid area. There has obviously been injury to the ... A. B. C. D. E. 48. superficial radial nerve deep radial nerve axillary nerve suprascapular nerve musculocutaneous nerve

In surgical exploration prior to removal of the gallbladder, it is often possible to find an aberrant hepatic artery to the right of the common bile duct. When it is aberrant, the right hepatic artery usually arises from the ... A. B. C. D. E. common hepatic artery left hepaticartery left gastric artery splenic artery superior mesenteric artery

49.

Volkmann's ischemic contracture is MOST OFTEN attributable to a ... A. B. C. D. E. fracture of the surgical neck of the humerus midshaft fracture of the humerus Colles' fracture of the wrist midshaft fracture of the radius supracondylar fracture of the humerus

50.

Which of the following is found normally in the urogenital diaphragm? A. B. C. D. E. bulbourethral glands of Cowper bulbospongiosus muscle Bartholin's gland Buck's basal bulbar fascia interureteric ridge or crest

51.

The peptic ulcer in the patient was observed with an endoscope and was seen to be located posteriorly in the first part of the duodenum. Bleeding into the intestinal lumen would be MOST LIKELY the result of erosion of the wall of the ... A. B. C. D. E. right gastroepiploic artery gastroduodenal artery dorsal gastric artery right gastric artery portal vein

52.

The 47 YO obese male patient complained to the family physician that he experiences severe abdominal pain after eating a heavy meal. The pain radiates to the area of the inferior angle of the scapula. Radiologic studies demonstrate clearly that the patient has numerous stones in the gallbladder. Referred pain from the gallbladder is due to afferent fibers that travel through the... A. B. C. D. E. phrenic nerve right vagus nerve lumbar splanchnic nerve 7th to 10th intercostal nerves greater thoracic splanchnic nerve

53.

When it is not possible to find superficial veins for insertion of a catheter in the upper limbs of an infant, it is usually possible to identify and utilize the great saphenous vein. This vessel is located... A. B. C. D. E. anterior to the lateral malleolus anterior to the medial malleolus posterior to the lateral malleolus posterior to the medial malleolus just posterior to the lateral femoral condyle

54.

The femoral artery is often used when it is necessary to thread a catheter into the heart. The artery is located within the femoral triangle of Scarpa. The lateral boundary of the femoral triangle is provided by the ... A. B. C. D. E. rectus femoris adductor longus pectineus sartorius iliopsoas

55. The 56 YO male had a pronounced bulge in the inguinal region, just lateral to the pubic tubercle. After the skin and superficial fascia had been incised and retracted and the external oblique aponeurosis was divided, the bulge could be seen clearly in the posterior wall of the inguinal canal, medial to the position of the deep inguinal ring, and therefore within the "triangle of Hesselbach." The lateral boundary of this triangle is the ... A. B. C. D. E. 56. border of the rectus abdominis muscle obliterated segment of the umbilical artery. lateral umbilical fold, containing the inferior epigastric vessels median umbilical fold iliacus muscle

The 17 YO male had been stabbed in the upper part of the abdomen. At laparotomy, with the abdomen opened, it is noted that profuse bleeding can be attributed to an injury to the liver, between the bed of the gallbladder and the falciform ligament. In an attempt to slow the bleeding, the Pringle maneuver is performed, but brisk arterial bleeding continues. The part of the liver that has been injured, AND the vessel which is primarily responsible for the bleeding are the . A. B. C. D. E. lateral segment of the left lobe, left hepatic artery medial segment of the left lobe, aberrant left hepatic artery caudate lobe of the liver, both the right and left hepatic arteries anterior segment of right lobe, aberrant right hepatic artery posterior segment of right lobe, right hepatic artery

57.

The 30 YO medical student was experiencing great pain in her abdomen; her abdominal wall was rigid, and the wall was virtually unmnoving in respiratory efforts. She admitted to drinking large quantities of coffee and caffeinated beverages in her attempts to stay alert in preparation for an upcoming series of exams. Which of the following DOES NOT TYPICALLY OCCUR in the event of either perforation or penetration of a duodenal ulcer? A. B. C. D. E. increase of borborygmi X-rav demonstration of air under the diaphragm paralytic ileus pain referral to the shoulder area bleeding from the gastroduodenal artery

58. In attempting to resect a tumor from the intestine, the surgeon noted the characteristic features of the specimen she removed. It possessed no taenia coli, nor appendices epiploica. Its internal foldings of mucosa were lush and extended well into the lumen. The mesentery attached was nearly transparent, and long vasa recti were evident within it. The organ, therefore, was... A. second part of the duodenum.

B. C. D. E. 59.

splenic flexure of colon ileum jejunum appendix

Caput medusa of the periumbilical region of a patient is often a characteristic associated with end-stage liver disease. These dilated vessels result from communication of anterior abdominal wall veins with periumbilical veins which accompany the ligamentum teres hepatis. The ligamentum teres hepatis closes and becomes ligamentous after birth. In addition, it can be said that it . A. B. C. D. E. is derived from the left umbilical vein is formed from the right umbilical artery is located within the dorsal mesogastrium carried deoxygenated blood while patent in the fetus carried blood from the liver to the inferior vena cava in the fetus

60.

The MOST COMMON type of hernia in adult women is the... A. B. C. D. E. femoral hernia direct inguinal hernia indirect inguinal hernia umbilical hernia epigastric hernia

61.

Because of the size of the baby, excessive traction on the head and neck was used to assist in its delivery from the birth canal. It was observed thereafter that the baby could not abduct, flex, nor laterally rotate the right arm; elbow flexion also seemed to be absent. The baby's hand seemed to function normally. What neural structure(s) was (were) injured at birth? A. B. C. D. E. the musculocutaneous and median nerves the lower trunk of the brachial plexus the posterior cord of the brachial plexus the upper trunk of brachial plexus, at Erb's point the ulnar, suprascapular and axillary nerves

62.

In blunt trauma to the male perineal or pelvic region, the urethra can be torn at the junction of its prostatic and membranous portions. In this event, into what space would extravasating (leaking) blood and urine be MOST LIKELY to flow? A. B. C. D. into the extraperitoneal. area in the pelvis into the urogenital diaphragm (deep pouch or deep compartment)

into the superficial perineal compartment (pouch) into a space between the fascia of Gallaudet and the superficial perineal musculature E. into the space between Campers fascia and Scarpas fascia of the abdominal wall

63.

The young man's hand was caught between the automobile and the pavement when the vehicle overturned. The tendons in the second compartment of the dorsum of his hand were torn severely. These tendons would include the... A. B. C. D. E. extensor pollicis longus and brevis extensor carpi radialis longus and brevis abductor pollicis longus and extensor pollicis brevis extensor digitorum and extensor digiti minimi extensor carpi ulnaris and extensor indicis

64.

Midline anterior abdominal wall incisions are used frequently because they provide rapid access to the contents of the abdominopelvic region, especially in emergency situations. The linea alba of the midline is ... A. B. C. D. E. vascularized richly by arteries of the area the site of insertion of the external and internal abdominal oblique muscles oriented parallel with the lines of Langer of the abdominal wall the site of the so-called Spigelian hernia characterized by decussation of fibers of the aponeuroses of the oblique and transverse muscles of the abdominal wall

65.

The posterior wall of the inguinal canal is a frequent site of herniation. It is formed PRIMARILY by... A. B. C. D. E. transversus abdominis aponeurosis and transversalis fascia transversalis fascia only, in the majority of people internal oblique muscle and its external and internal fascias lacunar ligament of Gimbernat external oblique tendon and fascia of Gallaudet

66.

A pheochromocytoma is a catecholamine secreting tumor which usually originates in the adrenal medulla. Which of the following can be said to be characteristic of the adrenal glands? A. The adrenal medulla is innervated by preganglionic sympathetic fibers. B. The right adrenal vein drains into the renal vein. C. The left adrenal gland receives all of its arterial supply from the left renal artery. D. The left adrenal gland is located within Morison's pouch, the deepest part of the abdominal peritoneal cavity when one is lying supine. E. The right vagus nerve (posterior vagal trunk) is the primary source of secretory fibers for release of norepinephrine by adrenal medullary cells.

67.

The 29 YO woman had been suffering increased difficulty in her morning run before class at the medical school. On examination, it was found that the muscle group of the medial side of her left thigh was obviously weaker and smaller than that on the contralateral limb. The conclusion was that her left obturator nerve was being compressed at the obturator foramen and required surgical release. Among the muscles that were suffering paralytic weakness would be the ...

A. B. C. D. E. 68.

rectus femoris gracilis vastus medialis tensor fasciae latae sartorius

In about one in 500 persons, the poles of the kidneys (usually the inferior poles) are fused, resulting in a horseshoe kidney. This congenital malformation is characterized further by the fact that... A. B. C. D. E. The fused kidneys remain in the pelvic cavity. In almost all cases the horseshoe kidneys are not functional. The ascent of the horseshoe kidney to a normal position is prevented by the root of the inferior mesenteric artery. The ureters do not enter the urinary bladder. The metanephric diverticulum does not develop.

69.

The 23 YO male was troubled by an enlarged testis. When he was seen by the university physician, a biopsy was taken and confirmed the tentative diagnosis of testicular cancer. The first lymph nodes to which the disease process would typically spread are the... A. B. C. D. E. superficial inguinal lymph nodes deep inguinal lymph nodes the node of Cloquet in the femoral canal the internal iliac nodes the pre- and paraaortic nodes at the level of the renal vessels

70.

The 60 YO woman had a marked lurch (a type of limp) when she walked. She would lean over her right hip to swing the left limb forward, thus balancing the weight of her upper body over the right lower limb. She could advance the right limb without severe difficulty. A very likely diagnosis in this case would be Trendelenburg gait. This dysfunction of ambulation results from weakness or paralysis principally of the ... A. B. C. D. E. gluteus medius gluteus maximus tensor fasciae latae "short lateral rotator group" of the gluteal region quadriceps femoris

71. A hard mass in the ostium of the patient's appendix had led to local infection appendicitis, with a slightly elevated temperature and moderate increase in WBC count. The initial pain from the infection was dull and difficult to localize, but the patient placed his hand in the periumbilical area to indicate the general area of discomfort. The region of the umbilicus receives its sensory supply, classically, from spinal nerve level ...

A. B. C. D. E. 72.

T7 T8 T10 T12 Ll

The painter had fallen from his scaffold for a considerable distance before managing to grasp a rope and arrest his fall. As a result, however, the lower trunk of the brachial plexus was injured because of the tension to the nerve trunk where it crossed over the first rib. Sensory deficits and motor paralysis resulted. Among the resultant problems, one should be able to anticipate A. B. C. D. E. paralysis of all interossei + loss of sensation over the lateral aspect of the dorsum of the hand inability to extend the elbow or MP joints + loss of sensation over the lateral aspect of the ventral surface of the forearm paralysis of opposition of thumb to other digits + absence of sensation over the medial aspect of the forearm and hand loss of ability to flex the elbow, whether supinated or pronated + anesthesia on the medial aspect of the dorsum of the hand All of the above

73.

The 24 YO patient, a medical student, presented in clinic complaining of burning pain and tingling sensations down the lateral aspects of both thighs. It was noted by the well trained nurse that the patient's bluejeans were very, very tight; in fact, it took considerable effort and a significant amount of time for the student to remove this article of clothing so that careful examination could be performed to exclude more serious diagnostic possibilities. Meralgia paresthetica often results from compression of a cutaneous nerve beneath the lateral extremity of the inguinal ligament. This nerve contains sensory fibers which are derived usually just from spinal nerve levels .... A. B. C. D. E. T12 L1 L1, L2 L2, L3 L2, L3 and L4

74. The 52 YO patient presented a complaint of impotence. Physical examination revealed atrophy of the musculature of his lower limbs and reduced pulses in the groin, popliteal fossa, ankles and feet. The patient admitted that his lower limbs tired easily. Gastrointestinal, renal and respiratory functions appeared within normal limits. The probable diagnosis in this case: A. B. C. D. E. 75. LeRiche syndrome, thrombus at the aortic bifurcation Embolic occlusion of-the "Critical Point of Sudeck". Aortic occlusive stenosis at the median arcuate ligament of the diaphragm Bilateral femoral arterial atherosclerotic occlusion Aortic aneurysm and intimal dissection proximal to renal arteries

Ultrasound studies revealed that the infant had congenital polycystic disease of the kidneys, causing the first year Pediatrics resident to review the embryologic development of these organs. The ureter, renal pelvis, calyces and collecting tubules develop as outgrowths from... A. B. C. D. E. the pronephric collecting system, including the nephrogenic cord the mesonephric tubules and proximal mesonephric duct the metanephric diverticulum the mesenchymal cells of the metanephric mesodermal mass (cap) the cranial ends of the paramesonephric ducts

76.

A 32 YO man had taken a fall when his motorcycle hit a rock in the road from adjacent sidewalk construction. He complains of numbness and tingling along the anterolateral surface of his leg and the dorsum of his foot. He cannot elevate his foot (dorsiflexion) nor extend his toes. He cannot evert the foot, either. X-Ray imaging reveals a fracture of the neck of the fibula. Which nerve has been injured in the accident? A. B. C. D. E. common fibular (peroneal) nerve deep fibular (peroneal) nerve tibial nerve saphenous nerve superficial peroneal (fibular) nerve

77.

Using a laparoscopic approach to the extrahepatic biliary system, the surgeon attempts to find the boundaries of the hepatocystic triangle of Calot, wherein the surgeon can isolate the cystic artery. The boundaries of this triangle include the... A. B. C. D. E. first part of duodenum, gastroduodenal artery and common bile duct splenic artery, neck of the pancreas and proper hepatic artery common hepatic duct, left hepatic artery, cystic duct liver surface, common hepatic duct, cystic duct common bile duct, cystic duct, visceral surface of the liver

78.

As he was examined in the clinic, it became obvious that the patient had several problems in his left leg. These included the following: anesthesia limited to the medial aspect of his leg; absence of his patellar reflex; weakness in dorsiflexion of his foot;near-total paralysis of inversion of the foot. The MOST LIKELY cause of this cluster of problems would be A. compression of the L4 spinal nerve root by disk herniation B. compression of the L5 spinal nerve root by disk herniation C. a tumor of the psoas sheath involving the lumbosacral trunk D. lesion of the sciatic nerve just proximal to the popliteal fossa E. compression of the tibial nerve between the gastrocnemius and soleus muscles

79.

The perineal body is incised rather often in parturition (childbirth). This serves to enlarge the vaginal opening, thus avoiding tearing of perineal tissues, especially to avoid tearing into the rectoanal canal. The perineal body ... A. B. C. D. E. is derived from fusion of the urorectal septum with the cloacal membrane. separates the anal membrane from the urogenital membrane provides the site of insertion of the superficial transverse perineus muscles provides the site of origin of the bulbospongiosus muscles is characterized by all of the above (A-D)

80.

The 44 YO female patient, a computer software engineer, pomplained that she was awakened frequently at night by pain in her right hand, particularly in the lateral portions of her hand, and the palmar surfaces of the thumb, index and long finger. Her right thenar eminence was reduced in size, compared with that of the contralateral hand. Among the structures that pass through the carpal tunnel, wherein the median nerve is often compressed, is the... A. B. C. D. E. superficial palmar branch of the radial artery deep ulnar nerve and deep branch of the ulnar artery deep palmar branch of the radial artery tendon of the flexor pollicis longus tendon of the palmaris longus

41.

A 21-year-old football player is referred to an orthopedic surgeon because of a fracture of the surgical neck of the humerus. Surgery reveals a hematoma that resulted from an artery that was torn by a bone fragment. This artery is MOST LIKELY the: A. B. C. D. E. Radial Axillary Brachial Profunda brachii (deep brachial) Posterior humeral circumflex

42.

A 50-year-old woman was recently diagnosed with carpal tunnel syndrome. Which of the following is LEAST LIKELY to be a predisposing factor in this condition? A. B. C. D. E. Dislocated lunate Thickened flexor retinaculum Tenosynovitis of long flexor tendons Compression of ulnar nerve Paresthesia or lack of sensation over thenar eminence

43.

Which of the following group of nerves is intimately related to a portion of the humerus and can be affected by fractures of the humerus? A. B. C. D. E. Axillary, musculocutaneous, radial Axillary, median, ulnar Axillary, radial, ulnar Axillarv, median, musculocutaneous Median, radial, ulnar

44. Inability to supinate the forearm could result from an injury to which of the following pairs of nerves? A. B. C. D. E. Suprascapular and axillary Musculocutaneous and median Axillary and radial Radial and musculocutaneous Median and ulnar

45.

A young mountain climber had fallen some distance before managing to grasp a safety line with one hand. The quick upward pull of the line on the weight of his body caused severe injury to the lower trunk of the brachial plexus. As a result of the lower trunk injury, one would CORRECTLY expect: A. B. C. D. E. Loss of function of the intrinsic muscles of the hand, with anesthesia along the medial aspect of his forearm and hand Loss of extension of the elbow and anesthesia on the dorsum of his arm and forearm Paralysis of the deltoid and the "SIT" group of muscles and anesthesia over the proximal, lateral side of the arm Inability to flex the elbow and anesthesia on the lateral side of the ventral surface of the forearm Paralysis of the serratus anterior and scapular winging, without sensory loss

46.

Which of the following is the CORRECT sequence from lateral to medial? A. B. C. D. E. Femoral vein, femoral artery, femoral ring, femoral nerve Femoral artery, femoral ring, femoral vein, femoral nerve Femoral artery, femoral nerve, femoral vein, femoral ring Femoral ring, femoral vein, femoral artery, femoral nerve Femoral nerve, femoral artery, femoral vein, femoral ring

47. Following an infection in the left hallux, a patient developed pain in the left groin. Likely reason for the pain is A. B. C. D. E. 48. referral of pain via the femoral nerve referral of pain via the saphenous nerve referral of pain via the obturator nerve lymphatic spread from the foot venous spread from the foot

To avoid damaging the sciatic nerve during an intramuscular injection in the right gluteal region, the clinician should insert the needle A. B. C. D. E. in the area over the sacrospinous ligament midway between the ischial tuberosity and the lesser trochanter at the midpoint of the gemelli muscles in the upper right outer quadrant of the gluteal region in the lower right quadrant of the gluteal region

49. A patient walks with a waddling gait that is characterized by the pelvis falling toward the side of the raised foot. Which of the following nerves is damaged? A. B. C. D. E. 50. Obturator nerve Nerve to obturator internus Superior gluteal nerve Inferior gluteal nerve Femoral nerve

An industrial accident produced a fracture of the upper third of the shaft of the femur. The proximal fragment was abducted and laterally rotated. Which of the following muscles was NOT INVOLVED in producing the above? A. B. C. D. E. Piriformis Obturator internus Iliopsoas Gluteus minimus Gluteus medius

51. is:

A convenient site for insertion of an intravenous line in the great saphenous vein

A. B. C. D. E. 52.

Anterior to the medial malleolus Inferior to the inguinal ligament Posterior to the lateral malleolus Medial to the popliteal vein Dorsal venous arch of foot

A patient presented with swelling of the entire left lower limb due to venous occlusion. The pelvic viscera remained normally drained. Likely site of obstruction will be: A. B. C. D. E. External iliac vein Common iliac vein Popliteal vein Inferior vena cava Great saphenous vein

53. A typical place for safe placement to insert the needle for performing lumbar puncture is A. B. C. D. E. at the level of vertebral level T12 - Ll at the level of a line drawn between the iliac crests at the level of a lire drawn between the posterior superior iliac spines at the level of the atlantoaxial articulation at the level of the sacral hiatus

54. A football player had been hit hard on the lateral side of his knee, with the ipsilateral foot planted on the ground and bearing his full weight, resulting in tearing of three structures in the joint. The "anterior drawer sign" was present upon examination. The structures injured would therefore MOST LIKELY include the: A. B. C. D. E. Fibular collateral and posterior cruciate ligaments Medial collateral and anterior cruciate ligaments Posterior cruciate ligament and lateral meniscus Lateral meniscus and fibular collateral ligament Ligamentum patellae

55. During an annual medical examination the physician gently passed a feather over the patient's forehead, nose (dorsum) and upper eyelid. The patient, however, was unable to feel a touch sensation. Which nerve carries the sensory fibers involved in the innervation of the skin of the forehead, dorsum of nose and upper eyelid? A. B. C. D. E. 56. Facial nerve Cervical spinal nerves Mandibular division of trigeminal Maxillary division of trigeminal Ophthalmic division of trigeminal

Following a traffic accident, a patient had difficulty chewing. She developed sensory loss from the mandibular teeth, anterior two thirds of tongue, lateral scalp and chin. The fracture site MOST LIKELY involved is the: A. B. C. D. E. Jugular foramen Internal acoustic meatus Foramen rotundum Carotid canal Foramen ovale

57.

In infants, chronic subdural hematoma can cause an enlarged head circumference. A subdural hematoma is MOST LIKELY to result from a ruptured A. B. C. D. E. middle meningeal artery middle meningeal vein cerebral artery cerebral vein cerebellar artery

58.

A 32-year-old woman sustained a head injury during a traffic accident, which resulted in palsy of her fourth cranial nerve. Injury to the 4th cranial nerve is MOST LIKELY to cause weakness of the: A. B. C. D. E. Superior oblique muscle Inferior oblique muscle Inferior rectus muscle Superior rectus muscle Lateral rectus muscle

59. A surgeon, at abdominal exploration, found dead intestinal segments (gangrene) involving the splenic flexure of the colon. He should expect to find a lesion or blockage in the: A. Ileocolic artery B. Middle colic artery C. Right colic artery D. Superior mesenteric artery or its branches E. Inferior mesenteric artery or its branches

60.

A 32-year-old man fell from his motorbike and received a hard blow to the lateral side of the upper part of his leg. He complains of numbness and tingling along the lower anterolateral surface of his leg and the dorsum of his foot. X-ray reveals a fracture of the neck of the fibula. Physical examination shows that he is unable to dorsiflex his foot. Which nerve is MOST LIKELY to be injured by a fracture of the neck of the fibula? A. B. C. D. E. Tibial nerve Lateral sural nerve Deep fibular (peroneal) nerve Saphenous nerve Common fibular (peroneal) nerve

61. A 25-year-old male patient who regularly exercises and works out with free weights in his health club comes to his physician complaining of a bulge in his groin. The patient shows Cruveilhier's sign. The physician notes a mass that protrudes at the external inguinal ring superior to the inguinal ligament and that extends medial to the pubic tubercle. The patient has a/an A. B. C. D. E. femoral hernia direct inguinal hernia torn iliopsoas muscle tendon enlargement of superficial inguinal lymph nodes (of Cloquet) obturator hernia

62. A male infant is born with several loops of intestine enclosed in a sac of peritoneum that protrudes through a defect in the abdominal wall. The defect is midline, but it lies outside the limits of the umbilical cord. The condition is A. B. C. D. E. gastroschisis omphalocele hiatal hernia exstrophy umbilical hernia

63. From the internal aspect of the abdominal wall, which of the following is the strongest area and LEAST prone to herniation? A. Lateral to the rectus sheath and inferomedial to the inferior epigastric vessels B. Below the inguinal ligament medial to the femoral vein C. The epigastric region above the umbilicus in the midline D. At the linea semilunaris (of Spieghel) E. Below the arcuate line of the rectus sheath in the midline 64. The median sacral artery A. B. C. D. E. 65. is a derivative of an embryological mesonephric artery supplies part of the lower sigmoid colon and rectum does not supply blood past the critical point of Sudeck is the main blood supply to the anorectal canal is a major site of thrombus formation affecting the sigmoid colon

Which section of the fetal blood circulation does NOT close off at birth? A. B. C. D. E. Umbilical arteries inferior to the allantois Umbilical arteries superior to the allantois Umbilical vein Ductus venosus Foramen ovale

66.

A surgeon is beginning a cholecystectomy (removal of the gall bladder) and in dissecting out and identifying the structures within cystic triangle (of Calot), she notes that there is an aberrant left hepatic artery. In approximately 20% of case the left hepatic artery derives from the A. B. C. D. E. gastroduodenal artery superior mesenteric artery right gastroepiploic artery posterior superior pancreaticoduodenal artery left gastric artery

67.

A 71-year-old female patient who a week previously suffered a myocardial infarction, presents with abdominal pain and symptoms of small intestinal blockage (ileus). Her physician determines that a thromboembolism has lodged in a vessel causing ischemia in a section of the ileum. In what blood vessel(s) is the embolism MOST LIKELY located? A. B. C. D. F. Ileocolic artery Ileal arcuate artery Superior mesenteric vein Ileal vasa recta Ileal veins

68.

The ureter, renal artery, and renal vein pass into and out of the kidney at the A. B. C. D. E. sinus lienorenal ligament hilum superior pole pelvis

69. An 8-day-old female infant shows marked abdominal distension and she has not defecated since birth. CT imaging reveals that the colon is enlarged and the rectum is constricted although the anus is perforate. External genitalia show no developmental abnormality. The physician diagnoses "congenital megacolon" (Hirshsprung's Disease). Anatomical defects in which of the following explain this condition? A. B. C. D. E. Enteric nervous system Parasympathetic nervous system Sympathetic nervous system Vertebral spinal segments L4-S4 Vertebral spinal segments S1-Co1

70. A 50-year-old male patient visits his physician because he has noticed a painful swelling in his left scrotum. The patient reports no history of trauma to the area. The physician examines the freely movable mass and determines that it is likely a varicocele. He suspects, however, abdominopelvic venous obstruction and orders an MRI image of the patient. A mass is indeed discovered and it is pressing and partially occluding the A. B. C. D. E. inferior vena cava at the level of L4 ascending lumbar veins on the right side left inferior epigastric vein left renal vein left hemiazygos vein

71. A 12-year-old boy attempts a handstand over the handlebars of his bicycle, falls onto the cross bar, and sustains a straddle injury. By the time he is taken to the emergency room blood from his injury has extravasated into the abdominal wall up to the level of the umbilicus. Under which layer of the abdominal wall has the blood collected? A. B. C. D. E. 72. Superficial layer of superficial fascia (Camper's fascia) External oblique fascia Anterior lamina of the rectus sheath Transversalis fascia Deep layer of superficial fascia (Scarpas fascia)

An odorless, cloudy fluid begins to exude from the umbilical stump of a 4-day-old infant. The fluid is MOST LIKELY derived from A. B. C. D. E. the urinary bladder the ileum the duodenum the colon a patent medial umbilical ligament

73.

A 21-year-old woman who was 10 weeks pregnant collapses at work and is rushed to the emergency room. Once there she is found to have internal bleeding and is in shock due to a ruptured intraperitoneal attachment site of an ectopic pregnancy. The embryos ectopic site of attachment was determined to be at the lowest point of the peritoneal cavity in erect position. This is known as the A. B. C. D. E. rectouterine pouch (of Douglas) hepatorenal pouch (of Morrison) lower recess of the omental bursa uterovesical pouch left lateral paracolic gutter

74. A surgeon is to remove the spleen in a patient who is suffering from a hereditary blood dyscrasia. To ligate the spleen's blood supply he first must locate the A. B. C. D. E. lesser omentum splenorenal ligament phrenicocolic ligament greater omentum hepatoduodenal ligament

75. A 52-year-old man who underwent prostate surgery three months previously is back in his physician's office because he now finds it impossible to have sexual intercourse because of erectile dysfunction. What nerve(s) was (were) severed in the procedure accounting for his problem? A. B. C. D. E. Pelvic splanchnic nerves Least splanchnic nerve Colic branches of the vagus nerve Pudendal nerve Prostatic plexus

76. A 35-year-old woman who has had four children all by vaginal delivery sees her doctor because she is partially incontinent. Any time her bladder becomes full she is at risk for involuntarily passing urine if she laughs or strains. In which of the following is this patient's problem located? A. B. C. D. E. 77. Pelvic diaphragm Superificial perineal pouch Deep perineal pouch Ureter Bladder

A 64-year-old man who has smoked 2 packs of cigarettes a day for the past 45 years and boasts of his robust good health is forced to see his doctor when he notices bright red blood in his stools. He denies any pain or discomfort but is now seriously concerned about his health. What is the MOST LIKELY arterial supply and the location of the lesion causing this patient's bleeding? A. B. C. D. E. Superior rectal artery; sigmoid colon Superior rectal artery; superior rectum Middle rectal artery; inferior rectum Inferior rectal artery; anus above the pectinate line (of Hilton) Inferior rectal artery; anus below pectinate line (of Hilton)

78.

A 54-year-old post-menopausal woman goes to her gynecologist complaining of dyspareunia (pain during sexual intercourse) and a lack of vaginal lubrication despite sexual arousal. This problem MOST LIKELY involves dysfunction of: A. B. C. D. E. Corpora cavernosa clitoridis Greater and lesser vestibular glands Gartner's duct Bulbospongiosus muscles Crus clitoridis

79.

The MAJOR ligamentous support(s) for the uterus is (are) the A. B. C. D. E. round ligament broad ligament suspensory ligament of the ovary and round ligament ovarian ligament uterosacral and cardinal ligaments

80.

Which of the following is NOT contributory to the pain of childbirth? A. B. C. D. E. Pelvic splanchnic nerves llioinguinal nerve Perineal branch of the posterior cutaneous nerve of the thigh Pudendal nerve Uterovaginal plexus

41.

During surgery a surgeon mistakenly clamps the splenic vein near the hilum of the spleen instead of the splenic artery in a patient undergoing splenectomy for an enlarged spleen. Using your knowledge of the structure and vascular supply of the spleen, what would be the result if this error is NOT immediately corrected? A. B. C. D. E. Retrograde flow into the inferior mesenteric vein Rupture and hemorrhage of the spleen Significant blood flow to the left renal vein Significant blood flow to anastomotic channels in the body wall Ischemia of the splenic parenchyma

42.

A 4-year-old presents to you with a fluctuant swelling in the left scrotum. On examination you can get above it and it transilluminates brilliantly. You decide to aspirate it. Which of the following structures will your needle NOT have to penetrate? A. B. C. D. E. Cremasteric fascia Tunica albuginea Dartos muscle Colles fascia External spermatic fascia

43.

During surgery to remove a patient's gall bladder, a surgeon locates the hepatocystic Triangle of Calot, finds the cystic artery, and then attempts to locate the hepatic arteries. He notes the left hepatic artery arising from the proper hepatic artery, but search as he might he fails to locate the right hepatic artery. Where does an aberrant right hepatic artery originate in approximately 20% of cases, and where should the surgeon now look to find this artery? A. B. C. D. E. Posterior superior pancreatoduodenal artery; posterior to the first part of the duodenum Right gastric artery; lesser curvature of the stomach Gastroduodenal artery; posterior to the first part of the duodenum Right inferior phrenic artery; superior to the hepatocystic triangle Superior mesenteric artery; posterior to the portal vein

44.

A visibly pumping vessel accounting for a bleeding peptic ulcer in the first part of the duodenum is MOST LIKELY the A. B. C. D. E. left gastroepiploic artery left gastric artery right gastroepiploic artery gastroduodenal artery right gastric artery

45.

The point in the colon where blood supply may become compromised during conditions of low blood pressure is

A. B. C. D. E. 46.

distal portions of the right colic artery proximal portions of the middle colic artery near the right colic flexure the critical point of Sudeck the terminal branches of the ileocolic artery, especially posterior cecal and appendicular branches the left colic flexure between terminal branches of the superior and inferior mesenteric arteries

A 55-year-old patient complaining of intense pain in his upper abdominal (epigastric) region is found to be suffering from acute pancreatitis. If the infection erodes the anterior peritoneal covering of the pancreas into which space will the exudate initially flow? A. B. C. D. E. Omental bursa (lesser sac) Left lateral paracolic gutter Hepatorenal recess (Morrison's Pouch) Right medial paracolic gutter Rectouterine pouch (Douglas's Pouch)

47.

A 43-year-old female comes to the Emergency Department complaining of extreme pain in the right upper quadrant of her abdomen. She has a history of gallstones and diagnostic imaging reveals a stone lodged in the most constricted portion of the biliary system. The stone is located at A. B. C. D. E. the sphincter of the bile duct minor duodenal papilla major duodenal papilla the distal end of the hepatopancreatic ampulla (of Vater) infundibulum of the gall bladder (Hartmann's Pouch)

48.

In congenital constriction of the duodenum caused by an anular pancreas, what embryonic primordium forms the constricting tissue? A. B. C. D. E. Ventral pancreatic bud Dorsal pancreatic bud Accessory pancreatic duct (of Santorini) Main pancreatic duct (of Wirsung) Terminal portion of the bile duct

49.

A section of gastrointestinal wall is sent from surgery to the pathology laboratory for testing. The macroscopic characteristics of the internal side of the specimen include large and numerous plicae circulares, and a velvety texture. In a portion of attached mesentery there are long vasa recta. The specimen is a portion of the A. B. C. D. ileum stomach duodenum jejunum

E. 50.

colon

Which of the following sections of the gastrointestinal system is attached to the body wall by a free mesentery, i.e. is "intraperitoneal"? A. B. C. D. E. Ascending colon Transverse colon Descending colon Third part of the duodenum Rectum

51.

Cranial nerve X supplies parasympathetic innervation to the gastrointestinal system as far as the left colic flexure. What is the source of parasympathetic supply to the GI tract from the left colic flexure to the anus? A. B. C. D. E. Coccygeal nerves Pudendal nerves Lesser splanchnic nerves Least splanchnic nerves Pelvic splanchnic nerves

52.

A 32-year-old man who comes into his physician's office complaining of pain in his left neck region is diagnosed with a duodenal ulcer. What nerve supply explains this pattern of referred pain? A. B. C. D. E. Posterior vagal trunk Left greater splanchnic nerve Left phrenic nerve Anterior vagal trunk Left long thoracic nerve

53.

Which of the following constitutes the primary ligamentous support of the uterus? A. B. C. D. E. Broad ligament Cardinal and uterosacral ligaments Pelvic diaphragm Round ligament Infundibulopelvic ligament

54.

An oncologist wishes to assess whether ovarian cancer in a 42year-old female patient has metastasized. Which group of lymph nodes should she check first? A. B. C. D. E. Common iliac Lumbar Sacral Deep inguinal External iliac

55.

Which of the following contribute(s) to the pain of childbirth? A. B. C. D. E. Lumbar splanchnic nerves Pudendal nerve Pelvic splanchnic nerves Ilioinguinal nerve All of the above

56.

A 61-year-old male patient has experienced constipation and has occasional bright red blood in his stool. He reports no pain except some discomfort from the constipation. He has no history of hemorrhoids. His physician suspects a tumor in the wall of the lower gastrointestinal tract. Where is the presumptive tumor located? A. B. C. D. E. In the descending colon At the external anal sphincter Above the pectinate line Below the white line in the anal canal In the transverse colon

57.

A 10-year-old boy is playing at a construction site when he falls backward onto a nail protruding from a piece of wood which pierces into the medial half of his right gluteal region. In the Emergency Department he is found to have intact thigh extension, knee flexion, and foot movement, but when he tries to walk his hip collapses toward the left side when he takes a step with his left leg. What nerve was damaged? A. B. C. D. E. Superior cluneal nerve Inferior gluteal nerve Sciatic nerve Superior gluteal nerve Inferior cluneal nerve

58.

An 18-year-old woman with no previous history of pregnancy is brought into the Emergency Department in shock and unconscious. It is determined that she is bleeding internally and a large volume of blood has accumulated in the peritoneal cavity. The probable cause is a ruptured arterial supply to an ectopic pregnancy. In which of the following locations of the peritoneal cavity is an ectopic pregnancy MOST LIKELY to lodge? A. B. C. D. E. Rectouterine pouch (of Douglas) Hepatorenal recess (Morrison's Pouch) Broad ligament Suspensory ligament of the ovary Inguinal canal

59.

A 58-year-old male successfully undergoes prostate surgery in order to correct the effects of prostatic hypertrophy, but he returns to his physician several months later complaining of erectile dysfunction. What nerves were severed in the surgery? A. B. C. D. E. Sympathetic fibers from the inferior hypogastric plexus Perineal branches of the posterior cutaneous nerve of the thigh Posterior scrotal nerves Parasympathetic fibers in the prostatic plexus Perineal branches of the pudendal nerve

60.

A post-menopausal woman sees her gynecologist because she has noticed what seems to be a mass in her vaginal wall. She has had three pregnancies with vaginal delivery and has had episiotomies in all three cases. Upon examination, the mass is found to be the cervix prolapsing into the vagina due to structural weakness of the pelvic floor and perineal body. The gynecologist prescribes "Kegel exercises" as a first measure to strengthen which component of the pelvic floor? A. B. C. D. E. Levator ani muscle Bulbocavernosus muscle Transversus perinei profundus muscle Transversus perinei superficialis muscle Sphincter ani internus

61.

A 12-year-old female gymnastics student is brought into the Emergency Department with a "straddle injury". She was engaged in beam exercises when she slipped and fell forcibly, striking her pubic bone on the bar. She has pain at the site of injury and significant swelling due to pooling blood in her lower abdominal wall up to the umbilicus. What space has the extravasated blood come to occupy? A. B. C. D. E. Between deep membranous (Scarpals) and deep muscular (Gallaudet's) fascia Deep perineal pouch Superficial perineal pouch Between deep membranous (Scarpals) and superficial fatty (Camper's) fascia Between transversalis and deep muscular (Gallaudet's) fascia

62.

A 26-year-old male construction worker visits his internist for a swelling in his groin. Physical examination determines that the swelling is a direct inguinal hernia. What layers has the herniated intestine traversed to exit at the superficial inguinal ring? A. B. C. D. E. Parietal and visceral peritoneum Parietal peritoneum and transversalis fascia Parietal peritoneum, and external oblique, internal oblique, and transversalis fascias Parietal peritoneum None; the hernial mass passes through the uninterrupted layers of the inguinal canal

63.

A 43-year-old long-distance truck driver visits his physician because he has painful hemorrhoids. Which vessels are responsible for Lemorrhoid formation? A. B. C. D. E. Internal pudendal and inferior epigastric veins Inferior and superior rectal veins Inferior and superior rectal arteries Internal pudendal and internal iliac arteries Prostatic plexus and inferior vesical veins

64.

During an emergency hysterectomy performed due to a hemorrhaging uterus following cesarean childbirth, a surgeon mistakenly clamps an elongated structure crossing the pelvic brim and passing under the uterine artery. This structure is A. B. C. D. E. ovarian ligament the round ligament the ovarian artery infundibulopelvic ligament the ureter

65.

Which of the following statements is INCORRECT?

A.

The superior laryngeal artery branches from the superior thyroid and travels with the internal laryngeal nerve B. The central artery of the retina travels in the optic nerve C. Rupture of the middle meningeal artery causes a subdural hematoma D. The vertebral artery ascends in the neck through the foramina transversaria E. The sphenopalatine artery passes from the pterygopalatine fossa into the nasal cavity through the sphenopalatine foramen 66. A 35-year-old man comes to your office following a neck injury, which involves the cervical portion of the sympathetic trunk. Physical examination leads to a diagnosis of Horner's syndrome. Which of the following are NOT clinical signs of Horner's syndrome? A. B. C. D. E. 67. Absence of the blink reflex Constriction of the pupil Drooping of upper eyelid (ptosis) Absence of facial sweating Redness of facial skin (flushing)

A blink reflex occurs in your patient when you lightly touch the cornea. Which nerve carries the sensory (afferent) fibers involved in this reflex? A. B. C. D. E. Optic nerve ophthalamic division of trigeminal nerve Maxillary division of trigeminal nerve Mandibular division of trigeminal nerve Oculomotor nerve

68.

Following a road accident, a skull x-ray reveals a fracture through the base of the skull involving the jugular foramen. Which of the following would NOT be seen with this type of injury? A. B. C. D. E. Lacrimal gland dysfunction Hoarseness Dysphagia (difficulty in swallowing) Parotid gland dysfunction Dropped shoulder on the same side

69.

A patient presents with an arrow embedded in the carotid triangle. Which of the following is NOT LIKELY to be involved? A. B. C. D. E. Brachial plexus Cervical sympathetic trunk Internal jugular vein Vagus nerve Carotid arteries

70.

Which of the following muscles would NOT be affected by compression of the median nerve in the carpal tunnel? A. B. C. D. E. Abductor pollicis brevis Second lumbrical Opponens pollicis Flexor pollicis brevis Adductor pollicis

71.

A large splinter, deeply embedded adjacent to the nail on the medial side of the ring finger develops into an abscess. Prior to lancing to remove the fragment and drain the abscess, the surgeon blocked with anesthetic the appropriate: A. B. C. D. E. Digital branch of the median nerve Superficial branch of the radial nerve Digital branch of the ulnar nerve Digital branch of the radial nerve Anterior interosseous branch of the median nerve

72.

An example of a nerve innervating a group of related muscles is: A. B. C. D. E. Obturator nerve to obturator internus and obturator externus Femoral nerve to adductors of thigh Sciatic nerve to gluteal muscle Superior gluteal nerve to lateral rotators of hip Tibial nerve to all intrinsic muscles on sole of the foot

73.

If the entire greater tubercle of the humerus was broken away as a result of an injury, which of the following movements of the humerus would be affected? A. B. C. D. E. Extension and medial rotation Flexion and abduction Abduction and lateral rotation Flexion and medial rotation Flexion, abduction and adduction

74.

A policeman was stabbed in the antero-medial third of the left thigh. At exploration, the knife was found to pass through the middle of the sartorius into the adductor canal. Which of the following is likely to be entered first in the path of the knife? A. B. C. D. E. The femoral artery The profunda femoris artery The profunda femoris vein The femoral vein The popliteal artery

75.

Inability to hold a piece of paper between the second and third digits would result from damage to the:

A. B. C. D. E. 76.

Ulnar nerve Anterior interosseous nerve Median nerve Superficial radial nerve Radial nerve

Following a fracture of the neck of the fibula, a patient noticed that he could dorsiflex but not evert his foot. Which of the following nerves was injured? A. B. C. D. E. The superficial peroneal (fibular) The common peroneal (fibular) nerve The deep peroneal (fibular) nerve The sural nerve Tibial nerve

77.

A tourist accidentally swallowed a bone while eating and the bone eventually got impacted in the anal canal below the dentate line. He suffered severe pain from this impaction. The pain experienced is mediated via: A. B. C. D. E. The inferior hemorrhoidal (rectal) nerves The superior hypogastric nerve The inferior hypogastric nerve The pelvic splanchnic nerve The vagus nerve

78.

Following an infection in the left hallus, (big toe), a patient developed pain in the left groin. Likely reason for the pain is: A. B. C. D. E. Referral of pain via the obturator nerve Lymphatic spread from the leg Referral of pain via the femoral nerve Referral of pain via the saphenous nerve Venous spread from the leg

79.

A 29-year-old female presenting with a mass in the right labia majora is LEAST LIKELY to have a/an: A. B. C. D. E. Direct inguinal hernia Skenes abscess Hematoma Bartholins abscess Indirect inguinal hernia

80.

A 29-year-old female presents with colicky abdominal pain associated with anorexia, nausea and vomiting. She has a tender lump palpable at the right inguinal ligament. The MOST LIKELY diagnosis is: A. B. C. D. E. Incarcerated direct inguinal hernia Femoral hernia (incarcerated) Femoral hernia (strangulated) Incarcerated indirect inguinal hernia Richter's hernia

1.

A 70-year-old man has difficulty in standing from a seated position but no difficulty in flexing or extending his leg. Which of the following muscles has been injured? A. B. C. D. E. Gluteus minimus and medius Hamstrings Adductor longus and brevis Gluteus maximus Quadriceps femoris

2.

In angina pectoris, the pain radiating down the left arm is associated with increased activity of sensory (afferent) fibers that travel along sympathetic pathways to reach the: A. B. C. D. E. Upper four or five cervical spinal cord segments Upper four or five thoracic spinal cord segments Upper two lumbar spinal cord segments Brain stem None of the choices are correct

3.

Which of the following groups of nerves is intimately related to a portion of the humerus and is MOST LIKELY to be affected by fractures of the humerus? A. B. C. D. E. 4. Axillary, musculocutaneous, radial Axillary, median, ulnar Axillary, radial, ulnar Axillary, median, musculocutaneous Median, radial, ulnar

The pampiniform plexus of veins on the right side empties into the: A. B. C. D. E. Inferior vena cava Right renal vein Inferior mesenteric vein Left renal vein Internal iliac vein

5.

A lesion to which one of the following structures at the location given would stop secretion from the lacrimal gland? A. B. C. D. E. Glossopharyngeal nerve at the jugular foramen Otic parasympathetic ganglion in the infratemporal fossa Greater petrosal nerve in the petrous temporal bone Oculomotor nerve in the superior orbital fissure Ciliary parasympathetic ganglion in the orbit

6.

A 70-year-old man visited his doctor after noticing his own voice sounded disturbingly loud (hyperacusis). Further examination revealed other symptoms such as inability to produce tears, impaired salivation, loss of taste from the anterior two thirds of his tongue and paralysis of the buccinator, platysma and orbicularis oris muscles. A

diagnosis of facial nerve lesion was made. At what level has the facial nerve lesion MOST LIKELY occurred? A. B. C. D. E. 7. At the level of the stylomastoid foramen Within the facial canal, just distal to the geniculate ganglion Within the facial canal, proximal to the origin of the chorda tympani nerve Within the facial canal, distal to the origin of the nerve to stapedius At the level of the internal acoustic meatus

A 70-year-old man develops a tumor in the distal part of the scalenus anterior muscle with pressure on related structures. Which of the following is LEAST LIKELY? A. B. C. D. E. Prominent suprascapular vessels Prominent transverse cervical vessels Prominent circumflex scapular Diminished flow in axillary vessels Prominent thoraco acrominal vessels

8.

A one-week old baby presents with secretions draining from the umbilicus. The MOST LIKELY diagnosis is: A. B. C. D. E. A fistula involving Meckels diverticulum Omphalocele Intestinal stenosis Anular pancreas Gastroschisis

9.

A 4-day-old infant has not passed stool since coming home from the hospital. On rectal exam a large fecal mass is found in the colon and a large release of gas and feces follows the rectal exam. The MOST LIKELY condition is: A. B. C. D. E. Anal agenesis Meckels diverticulum Rectal atresia Colonic aganglionosis Volvulus of hindgut

10.

In surgery upon a broken femoral neck, it is critical to be aware of key anatomic features of the hip area. For instance, the "key" to the gluteal region is the A. B. C. D. E. tendon of the oburator internus sacrospinous ligament ischial spine site of intersection of the ilium and ischium in the acetabulum piriformis muscle

11.

A 24-year-old female patient received a small caliber bullet wound to the popliteal fossa from a drive by assailant (person with a gun in an auto). The bullet severed the tibial nerve. Such an injury would MOST LIKELY result in

A. B. C. D. E. 12.

inability to extend the leg at the knee foot drop a dorsiflexed and everted foot a plantar flexed and inverted foot total inability to flex the leg at the knee joint

Vessels which participate significantly in the cruciate ("crucial") anastomosis of the gluteal region include the A. B. C. D. E. superior gluteal artery descending branch of the lateral femoral circumflex artery ascending branch of the medial femoral circumflex artery deep external pudendal artery deep circumflex iliac artery

13.

Cranial nerve X supplies parasympathetic innervation to the gastrointestinal system as far as the left colic flexure. What is the source of parasympathetic supply to the GI tract from the left colic flexure to the anus? A. B. C. D. E. Lesser splanchnic nerves Least splanchnic nerves Coccygeal nerves Pudendal nerves Pelvic splanchnic nerves

14.

A 10-year-old boy is playing at a construction site when he falls backward onto a nail protruding from a piece of wood which pierces into the medial half of his right gluteal region. In the Emergency Department he is found to have intact thigh extension, knee flexion, and foot movement, but when he tries to walk his hip collapses towards the left side when he takes a step with his left leg. What nerve was damaged? A. B. C. D. E. Sciatic nerve Superior gluteal nerve Superior cluneal nerve Inferior gluteal nerve Inferior cluneal nerve

15.

An example of a nerve innervating a group of related muscles is: A. B. C. D. E. Sciatic nerve to gluteal muscle Superior gluteal nerve to lateral rotators of hip Obturator nerve to obturator internus and obturator externus Femoral nerve to adductors of thigh Tibial nerve to all intrinsic muscles on sole of the foot

16.

A policeman was stabbed in the antero-medial third of the left thigh. At exploration, the knife was found to pass through the middle of the sartorius into the adductor canal. Which of the following is likely to be entered first in the path of the knife? A. B. C. D. E. The femoral vein The femoral artery The profunda femoris artery The profunda femoris vein The popliteal artery

17.

Following a fracture of the neck of the fibula, a patient noticed that he could dorsiflex but NOT EVERT his foot. Which of the following nerves was injured? A. B. C. D. E. The common peroneal (fibular) nerve The deep peroneal (fibular) nerve The sural nerve The superficial peroneal (fibular) Tibial nerve

18.

Following an infection in the left hallus, (big toe), a patient developed pain in the left groin. MOST LIKELY reason for the pain is: A. B. C. D. E. Referral of pain via the femoral nerve Referral of pain via the saphenous nerve Referral of pain via the obturator nerve Lymphatic spread from the leg Venous spread from the leg

19.

A physician examines a 53-year-old man who has a history of alcoholism and who has come to the Emergency Room with multiple physical complaints. The physician notices that the man has a yellowish cast to his skin and that there is a radiating pattern of engorged veins ("caput medusae") on the surface of his abdomen. While in the examining room the patient suddenly vomits a copious amount of blood. Based on your knowledge of the portal venous system, which of the following conditions does the patient likely also have? A. B. C. D. E. Barett's esophagus Renal stones Atherosclerosis Hypertension Hemorrhoids

20.

A 62-year-old man comes into the Emergency Room complaining of a painful mass in his left testicle. Physical examination reveals a varicocele with a "bag of worms" feel in the left testes. The patient also evinces advanced atherosclerotic disease. The MOST LIKELY anatomical explanation for this patient's symptoms is: A. B. C. D. E. Compression of the left testicular vein by an aortic aneurysm Blockage of the left internal pudendal vein Compression of the left renal vein by the superior mesenteric artery Trauma to the left pampiniform plexus of veins "Bifurcation (Leriche) syndrome" with internal iliac artery blockage

21.

During surgical removal of the right kidney from the posterior aspect, a surgeon accidentally cuts a nerve deriving from the first lumbar segment and supplying the lower anterior abdominal wall. This nerve is the right A. B. C. D. E. subcostal ilioinguinal iliohypogastric iliolumbar femoral

22.

A 40-year-old female patient with a lump in her right breast is discovered to have carcinoma of the breast that has spread to the underlying deep fascia and pectoralis musculature. Based on the pattern of lymphatic drainage, which of the following locations is the MOST LIKELY site of metastasis? A. B. C. D. E. Liver Stomach Spleen Descending colon Uterus

23.

A 71-year-old female patient presents with upper abdominal pain that is traced to a blocked pancreatic duct causing an enlarged and inflamed head of the pancreas. Fluid from the enlarged pancreatic head pushes its anterior covering of visceral peritoneum into a so-called "pancreatic pseudocyst" which extends into which portion of the peritoneal cavity? A. B. C. D. E. Hepaticopancreatic recess ("Morrison's Pouch") Right lateral colic gutter of the greater sac Right medial colic gutter of the greater sac Left medial pancreatic recess of the greater sac Lesser sac (omental bursa)

24.

A surgical resident is performing a cholecystectomy and is attempting to locate and ligate the cystic artery within the hepatocystic triangle (of Calot). He finds the left hepatic artery entering the liver and traces it back to its origin from the common hepatic artery, in company with the gastroduodenal artery, but there is no sign of right hepatic artery originating from any branch of the celiac trunk. What is the MOST

probable origin in this case of the right hepatic artery and its important branch, the cystic artery? A. B. C. D. E. 25. Right inferior phrenic artery Right superior suprarenal artery Right colic artery Superior mesenteric artery Marginal artery (of Drummond)

An 18-year-old male victim of a motor vehicle accident is brought into the Emergency Room with crush injuries to his lower limbs and pelvis. Imaging shows that his pubis is fractured and that his prostatic urethra is torn. Blood and urine will enter which space? A. B. C. D. E. Deep perineal pouch Superficial perineal pouch Superficial perineal cleft Ischianal fossa Retropubic space (of Retzius)

26.

A section of gastrointestinal wall is sent from surgery to the pathology laboratory for testing. The macroscopic characteristics of the internal side of the specimen include large and numerous plicae circulares, and a velvety texture. In a portion of attached mesentery there are long vasa recta. The specimen is a portion of the A. B. C. D. E. stomach duodenum jejunum ileum colon

27.

Which of the following sections of the gastrointestinal system is attached to the body wall by a free mesentery, i.e. is "intraperitoneal"? A. B. C. D. E. Third part of the duodenum Ascending colon Transverse colon Descending colon Rectum

28.

A 32-year-old man who comes into his physician's office complaining of pain in his left neck region is diagnosed with a duodenal ulcer. What nerve supply explains this pattern of referred pain? A. B. C. D. E. Left greater splanchnic nerve Left phrenic nerve Anterior vagal trunk Posterior vagal trunk Left long thoracic nerve

29.

Which of the following constitutes the primary ligamentous support of the uterus? A. B. C. D. E. Round ligament Broad ligament Cardinal and uterosacral ligaments Pelvic diaphragm Infundibulopelvic ligament

30. An oncologist wishes to assess whether ovarian cancer in a 42-year-old female patient has metastasized. Which group of lymph nodes should she check first? A. B. C. D. E. 31. Sacral Deep inguinal Common iliac Lumbar Superficial inguinal

Which of the following contribute(s) to the pain of childbirth? A. B. C. D. E. Pudendal nerve Pelvic splanchnic nerves Ilioinguinal nerve Lumbar splanchnic nerves All of the above

32.

A 61-year-old male patient has experienced constipation and has occasional bright red blood in his stool. He reports no pain except some discomfort from the constipation. He has no history of hemorrhoids. His physician suspects a tumor in the wall of the lower gastrointestinal tract. Where is the presumptive tumor MOST LIKELY located? A. B. C. D. E. Below the white line in the anal canal In the descending colon At the external anal sphincter Above the pectinate line in the anal canal In the transverse colon

33.

An 18-year-old woman with no previous, history of pregnancy is brought into the Emergency Department in shock and unconscious. It is determined that she is bleeding internally and a large volume of blood has accumulated in the peritoneal cavity. The probable cause is a ruptured arterial supply to an ectopic pregnancy. In which of the following locations of the peritoneal cavity is an ectopic pregnancy MOST LIKELY to lodge? A. B. C. D. E. Hepatorenal recess (Morrison's Pouch) Broad ligament Suspensory ligament of the ovary Rectouterine pouch (of Douglas) Inguinal canal

34.

A 58-year-old male successfully undergoes prostate surgery in order to correct the effects of prostatic hypertrophy, but he returns to his physician several months later complaining of erectile dysfunction. What nerves were severed in the surgery? A. B. C. D. E. Parasympathetic fibers in the prostatic plexus Sympathetic fibers from the inferior hypogastric plexus Perineal branches of the posterior cutaneous nerve of the thigh Posterior scrotal nerves Perineal branches of the pudendal nerve

35.

A post-menopausal woman sees her gynecologist because she has noticed what seems to be a mass in her vaginal wall. She has had three pregnancies with vaginal delivery and has had episiotomies in all three cases. Upon examination, the mass is found to be the cervix prolapsing into the vagina due to structural weakness of the pelvic floor and perineal body. The gynecologist prescribes "Kegel exercises" as a first measure to strengthen which component of the pelvic floor? A. B. C. D. E. Transversus perinei profundus muscle Transverses perinei superficialis muscle Levator ani muscle Bulbocavernosus muscle Sphincter ani internus muscle

36.

A 26-year-old male construction worker visits his internist for a swelling in his groin. Physical examination determines that the swelling is a direct inguinal hernia. What layers has the herniated intestine traversed to exit at the superficial inguinal ring? A. B. C. D. E. Parietal peritoneum Parietal and visceral peritoneum Parietal peritoneum and transversalis fascia Parietal peritoneum, and external oblique, internal oblique, and transversalis fascias None; the hernial mass passes through the uninterrupted layers of the inguinal canal

37.

A 43-year-old long-distance truck driver visits his physician because he has painful hemorrhoids. Which vessels are responsible for hemorrhoid formation? A. B. C. D. E. Inferior and superior rectal arteries Internal pudendal and internal iliac arteries Internal pudendal and inferior epigastric veins Inferior and superior rectal veins Prostatic plexus and inferior vesicle veins

38.

During an emergency hysterectomy, performed due to a hemorrhaging uterus following cesarean childbirth, a surgeon mistakenly clamps an elongated structure crossing the pelvic brim and passing under the uterine artery. This structure is A. B. C. D. E. the round ligament the ovarian artery infundibulopelvic ligament ovarian ligament the ureter

39.

When a person burns the tip of their tongue drinking boiling ho coffee, the pain impulse will be carried to neurons located in the A. B. C. D. E. geniculate ganglion submandibular ganglion semilunar (trigeminal) ganglion pterygopalatine ganglion inferior glossopharyngeal ganglion

40.

When performing Pringle's manouvre, which of the following statements is INCORRECT? A. B. C. D. E. The free edge of the lesser omentum is pinched Blood flow through the cystic artery is arrested It can be used during cholecystectomy The portal vein is compressed The left gastric artery is compressed

You might also like